This question speaks to the arguments that lead into the main idea of the passage. The passage begins with the author’s description of Cézanne’s attempt to paint objectively, only to shift in the second paragraph to a question: “let us ask why, in the long history of art, it had never previously happened that an artist should wish to see the world objectively.” The reason, the author argues, seems to be because of “the very nature of perception, which does not present to the senses a flat two-dimensional picture with precise boundaries but a central focus with a periphery of vaguely apprehended and seemingly distorted objects.” This is why “there always intervened between the visual event and the act of realizing the vision an activity which we can only call interpretive.” So the answer is going to have to say something about how interpretation is made necessary by the way we perceive things.
This is what Cézanne was trying to do, but the passage author actually never quite admits that we can interpret our way to objectivity. If anything, paragraph 3 seems to suggest the opposite: “perspective does not give us any glimpse of the reality.” A therefore has to be wrong.
B matches my prediction, since “act of seeing” here is synonymous with perception. I can choose B and move on.
C distorts the passage. Interpretation in painting is described as occurring well before Cézanne in paragraph 2, when the author states that Greek and Roman art, as well as Renaissance art, always involved interpretation.
The passage barely mentions what happens after Cézanne—in the very beginning of the passage, when it mentions that “the modern movement in art begins” with Cézanne—so I’m immediately suspicious of D. But D can’t be correct because interpretation is firmly associated with people who failed to do what Cézanne was trying to do. Interpretation was the dominant force in art many years before Cézanne; I don’t have enough passage information to determine whether it was also so dominant afterwards. That’s enough to make D incorrect.
This builds on the work I was forced to do with the previous question. The only added component here is that I need to account for the fact that the author begins and ends with Cézanne. If, as paragraph 1 states, Cézanne wanted “to see the world objectively,” and paragraphs 2 and 3 outline how much of a problem that is, the passage author returns to Cézanne in the end: stating that “he was familiar with the ‘art of the museums’ and respected the attempts of his predecessors to come to terms with nature,” but that he still tried to paint objectively. Since the previous arguments about how impossible it might be to paint objectively lead into this point about Cézanne still trying to do so, the right answer must be that Cézanne tried to paint his way to objectivity in spite of the problems inherent in relying on perception.
This is mentioned in paragraph 1, where it is a minor point: one that is just meant to contextualize why Cézanne would have wanted to try to paint objectively. Since A does not encapsulate either the arguments about Cézanne or the arguments about perception in the passage, A can’t be correct.
This is perhaps the most tempting answer choice, if only because it draws on ideas and concepts expounded on heavily in the middle of the passage. But B is incorrect because it distorts the passage. Paragraph 2 says that perspective is what made interpretation necessary; it therefore did not prevent artists from interpreting anything. B therefore has to be wrong.
C encapsulates all the major parts of the passage: the Cézanne arguments from the first and last paragraphs, and the problem of interpretation from paragraphs 2 and 3. That makes C correct.
D is only a part of the argument made in paragraph 2, but it is one that is both clearly in service to explaining Cézanne—since the point about older art is prefaced by the question of why no one else attempted to do what Cézanne attempted to do—and is a point that the passage builds on in order to argue that this desire still required artists to interpret reality. The right answer to this question type has to be where the main bulk of passage arguments lead to; since D leads to other passage arguments, it cannot be correct.
It’s almost certain that the passage author has to know that people might not get what the term “will-o’-the-wisp” means, so an explanation for it almost certainly has to immediately follow. And it does: “an actuality we can see but never grasp.” So that means that the author is suggesting that artists before Cézanne had tried and failed to represent reality. Before going to the answers, it’s important to note that the answer choices all have to be read negatively, since the question stem says “artists before Cézanne had not.” So I’m looking for an answer choice which says something like those artists did find a way to depict reality faithfully.
If anything, the reference to something that “we can see but never grasp” suggests the opposite of A: that artists tried to imitate reality faithfully, but failed. That’s in line with what the author argues in paragraphs 2 and 3 about past artists and their attempts to imitate the world while relying on interpretation. A therefore has to be wrong.
I’m immediately suspicious of B because the passage barely talks about Impressionism. What’s worse is that Impressionism is mentioned in paragraph 1, and the quote from the question stem comes from paragraph 4. But finally, B has to be wrong because there is no discussion in the passage about the value of Impressionism.
C is just flat-out wrong. The passage is about what artists were able to do with their art, not with their actual physical ability to perceive reality.
D is the last choice standing, and luckily it is correct. For the author to use a metaphor of trying to touch an illusion to describe artists’ attempts to depict reality is to say that artists never actually succeeded in those attempts.
Though the question stem doesn’t clearly suggest what kind of question it is, the answer choices tell me that the right answer will describe the part of the passage argument that the question stem references. In particular, the answer choices will all be descriptions of how the author has written the passage: what surrounds the quote, or relates to it. There’s no easy way to predict the right answer to this, since the answer choices can describe the passage in many different ways, but I can prepare to find the right answer by simply looking over the part of the passage the question stem asks about and remembering how it works in relation to the main idea. I know that the references to Greek, Roman and Renaissance art happens in paragraph 2, and it is part of the author’s arguments about how interpretation is a necessary part of representing reality. That should be enough to find the right answer.
There are no specific works of art mentioned anywhere in the passage, so A can’t be right.
This is correct. The author simply states that artists from these periods tried to represent the world accurately. There is no evidence before or after the claim to support it; before that claim, the author talks about Cézanne’s attempts to depict reality objectively, and after this claim the author shifts to talking about the problem painters from those eras and afterwards faced. Since B accurately depicts what happens in the passage, it has to be correct.
C actually inverts the argument in paragraph 2. The discussion of art from the Greek, Roman, and Renaissance periods supports the discussion of the nature of perception; it helps to set up that discussion by establishing that even artists who tried to depict the world “as it really is” had to interpret that world. C is therefore incorrect.
There are no contradictions in the passage, and the passage author never again discusses art from the periods mentioned in the question stem, so D can’t be correct.
I don’t really know much about the influence of Cézanne’s work based off of the passage. The first sentence is really my only clue: “There is no doubt that what we call the modern movement in art begins with the single-minded determination of a French painter to see the world objectively.” This aligns with the only thing I really about Cézanne from the passage: that he tried to paint the world objectively. So the right answer has to involve that.
A mentions objectivity, and it echoes the last sentence, which states that even though Cézanne knew of the failures of past artists who tried to paint the world objectively, he “did not despair of succeeding where they had failed-that is to say, in ‘realizing’ his sensations in the presence of nature.” Still, since A is the first answer, and since I didn’t have a full prediction for this question, it makes sense to eliminate the other answer choices before moving on.
“Extra-visual faculties” are mentioned in paragraph 3 as what artists did to solve the problem of perception, and the persistent difference between perception and reality. The passage does not say that Cézanne did this, though, so B has to be incorrect.
I don’t know anything about what Cézanne did with interpretation, so I’m immediately tempted to eliminate C. C also probably has to be wrong because the passage poses the problem of interpretation as one that Cézanne would have had to overcome in order to paint the world objectively. In either case, there just isn’t enough information with regards to Cézanne doing anything with interpretation for C to be correct.
D echoes the passage definition of the Impressionists, who Cézanne was trying to move away from according to paragraph 1, so D has to be incorrect.
The passage really plays up how Cézanne was the first person to really make a push to see the world objectively. That’s especially suggested by the second paragraph: to say “let us ask why, in the long history of art, it had never previously happened that an artist should wish to see the world objectively” is to imply that Cézanne is the first. So at the very least, the information in the question stem would challenge the idea that Cézanne was the first painter to attempt to paint objectively. That should be enough to find an answer choice.
The author never states that Cézanne tried to do what Greek and Roman art tried to do. If anything, the passage use of the term “imitative” in that paragraph, as opposed to the already established “objective” from paragraph 1 used to describe Cézanne’s approach, implies they had different goals. Since the passage never makes a contention like A, A can’t be correct.
B isn’t the answer that I was expecting, but it triggers my memory of the first sentence of the passage: “There is no doubt that what we call the modern movement in art begins with the single-minded determination of a French painter to see the world objectively.” I suppose the implication there is that whoever was the first person to attempt to paint the world objectively would have started the modern art movement, which could make B right. But I also don’t like that as a justification for B, since that first sentence doesn’t actually require Cézanne to be the first painter to attempt to paint objectively. Instead, that sentence brings up Cézanne’s “single-minded determination,” which is again raised in the last paragraph. It seems entirely possible that Cézanne could have gotten the idea of objective painting from someone else, but was either more successful at it or applied himself so fully to that task that it helped spark modern art. Since I have these reservations, I’m going to hold onto B, but also double check to see if any of the other answer choices are better.
I don’t see how Cézanne learning about objective art from someone else would suggest that Cézanne did not respect his artistic predecessors. One just doesn’t have anything to do with the other. So I can eliminate C.
D is actually the same answer choice as C. When the last paragraph states that Cézanne “was familiar with the ‘art of the museums,’” it does so in order to say that Cézanne “respected the attempts of his predecessors to come to terms with nature.” Since C and D are the same answer choice, both cannot be correct. That makes both wrong. B, therefore, has to be the right answer.
This question is testing if I understand a rhetorical move made by the passage author. This comparison is made in paragraph 3, which I know is meant to reinforce paragraph 2: to help drive home the idea that interpretation is an eternal roadblock to objective depictions of the world. The specific quote is “a system of perspective is no more an accurate representation of what the eye sees than a Mercator’s projection is what the world looks like from Sirius.” The author goes on to explain this comparison in the next sentence: “Like the map, it serves to guide the intellect; perspective does not give us any glimpse of the reality.” So I’m looking for an answer which refers to this sentence.
The entire point of the paragraph is to say that representations are not accurate depictions of the world by way of claiming that maps are also representations of the world, so the passage can’t be saying that maps are more accurate than anything else. A has to be wrong.
The author doesn’t talk about maps enough for B to be correct. B would require a point about the history of mapmaking, while the author just refers to a kind of map to make a very specific point about representation. B is therefore wrong.
C is the exact opposite of what the author says at the end of paragraph 3; neither the map nor the system of perspective “give us any glimpse of the reality.” C is therefore wrong.
D, however, matches the first half of the end of paragraph 3: that both maps and systems of perspective “guide the intellect.” That’s enough of a similarity for me to choose D and move on.
This is a strange question: one that asks me to apply passage logic to a new situation. To do this, I need to figure out what part of the passage could inform my understanding of the situation in the question stem. In this case, I need to go to a point where the author talks about the meaning of words in a painting. This happens in paragraphs 5, 6, and 7. Paragraph 5 gives the grammatical depiction of a phrase “Even in Arcady, there am I,” which the author states “had been consistent and easily intelligible as long as the words could be attributed to a death’s head and as long as the shepherds were suddenly and frighteningly interrupted in their walk.” But the changes Poussin makes, which are mentioned in paragraph 6, match the question stem: “When facing the Louvre painting, however, the beholder finds it difficult to accept the inscription in its literal, grammatically correct, significance” because of “the absence of a death’s head.” The author states that without that head, “the ego in the phrase might seem to refer to the tomb itself.” But it is infinitely more natural to ascribe the words to the person buried therein.” What the author seems to be suggesting, then, is that the painting itself offers clues regarding how to interpret the words. That should be enough to find a right answer.
I don’t have a good reason to choose A. The last paragraph suggests that Poussin is trying to change the meaning of the phrase, and not just confuse viewers, so A has to be incorrect.
The author does the opposite of B. The words are only meaningful because of the scene; the author only knows what to do with ego in the words on Poussin’s painting because of what the scene is depicting. It is not the case that the scene is read through the words: that the words are used to make the image make sense. Therefore, B has to be incorrect.
C matches my prediction, and matches the author’s own explanation of how to read words that have lost their original context. I can choose C and move on.
Since the passage spends multiple paragraphs trying to understand the words of Poussin’s painting, the author is not likely to want to study the scene without also discussing the words. That makes D incorrect.
The passage tells me that the phrase mentioned in the question stem is the original translation of the words: one that is truer to Latin grammar (paragraph 5), and is in line with Guercino’s painting. So it would be reasonable to assume that English viewers were drawing from their familiarity with Guercino’s painting when interpreting Poussin’s.
If anything, the situation in the question stem suggests that the English might have been pretty familiar with grammar, since the quote in the question stem is grammatically correct (unlike Poussin’s).
This is associated with Poussin, but the phrase in the question stem is not. The question stem implies that English viewers may not have been fully in tune with Poussin, and offers no evidence that they were in tune with what Poussin was doing. That’s enough to make B incorrect.
I don’t know anything about sophistication on the basis of the passage, so I can eliminate C. Even if I assume that Poussin’s painting was more sophisticated than Guercino’s because it built on and modified that painting, that would make the English less sophisticated viewers, not more, since they failed to pick up on the details in Poussin’s painting which hint at how to read the words.
D matches my prediction, and so I can choose it and move on.
The answer choices confirm that my job with this question is to determine if the new information in the question stem strengthens or weakens the passage argument, and to what extent. The answer choices will usually describe complete or partial strengtheners or weakeners. I know that the author’s thesis about the painting is that “Poussin’s Louvre picture no longer shows a dramatic encounter with Death but a contemplative absorption in the idea of morality” (paragraph 4). This is illustrated by a change in the way the Latin phrase in the painting is read: one that shifts the phrase from “Even in Arcady, there am I” to “I, too, lived in Arcady.” I also know this shift happens because of two changes that Poussin makes: the shepherds are no longer shocked by what they see, but are instead “absorbed in calm discussion and pensive contemplation,” and “the form of the tomb is simplified into a plain rectangular block, and the death’s-head is eliminated altogether.” That I’m supposed to focus on these two when understanding the author’s argument regarding how to translate the words in Poussin’s painting is reinforced in paragraphs 5 and 6, since the original translation made sense “as long as the words could be attributed to a death’s-head and as long as the shepherds were suddenly and frighteningly interrupted in their walk.” The question stem tells me that Poussin actually painted a death’s-head into this painting, so one of the two pieces of evidence the author uses to make the passage claim is weakened. But since there are two pieces of evidence—neither of which is described as more important than the other—and the information in the question stem does not touch on the second piece of evidence—Poussin’s changing the shepherds from surprised to calmly talking to each other—at least a part of the passage author’s argument still stands. So I know that the right answer will be that the information weakens the passage, but does not totally disprove it.
The information in the question stem contradicts a major part of the author’s argument about Poussin’s painting: that the death’s-head is absent. So there is no way that it would strengthen the author’s argument to the point of confirming it.
B is wrong for the same reasons as A: because B describes a strengthener, while the new information in the question stem would weaken the passage argument.
C matches my prediction, so I would choose it and move on.
D can’t be correct, because in order for the information in the question stem to disprove the author’s argument, it would have to directly contest all of the major evidence the author brings to bear to argue the main point. D would only be correct if the question stem also suggested that Poussin’s depiction of the shepherds also did not differ from Guercino’s painting. But since the question stem doesn’t do that, D has to be wrong.
This claim is made in paragraph 3, which describes the changes Poussin made in his second revision of Et in Arcadia ego. Even if I don’t necessarily know what the “medieval, moralizing tradition” is, I know that what follows has to describe what in the painting suggests it breaks with that tradition. So the right answers just have to be mentioned as changes Poussin made in paragraph 3.I. A classical tomb I is mentioned as a change made to Guercino’s original composition in paragraph 1, but it is the change from a classical tomb into a “simplified […] plain rectangular block” that paragraph 3 implies is part of the reason why the painting breaks with the medieval tradition.
II. A pagan river god II is just not mentioned at all in that paragraph, which is as good a reason as any for II to be correct. II is actually also listed as one of the changes Poussin made in his first revision of Guercino’s painting in paragraph 1, but since that version is not described as breaking with the medieval moralizing tradition, I don’t have enough information to know if that specific change says anything about the painting’s relationship to the medieval tradition.
III. A symmetrical composition III, though, is mentioned: “Instead of two or three Arcadians approaching from the left in a group, we have four, symmetrically arranged on either side of a sepulchral monument.” So the right answer has to just say III.
II is incorrect because I don’t see any mention of a pagan river god in paragraph 3. I don’t even know if the river god made it into the second version of Poussin’s painting, since the third paragraph only claims to discuss the changes which Poussin made that also challenged the medieval moralizing tradition. It is not necessarily a comprehensive list of all the changes Poussin made to both his earlier painting and Guercino’s.
B matches my prediction, so I can choose it and move on.
Both I and II are incorrect, so C has to be wrong.
D is wrong because the tomb is implied to be changed, which in turn suggests that it is not a classical tomb.
This is a simple passage check question, since I know the author explains this in paragraph 3: “Instead of being checked in their progress by an unexpected and terrifying phenomenon, they are absorbed in calm discussion and pensive contemplation.” It also helps to know that in paragraph 2, having the shepherds approach the tomb is said to keep some “element of drama and surprise” in common with Guercino’s painting. So I know I’m looking for a choice which says that the shepherds will not be surprised.
A matches my prediction, so I can choose it and move on.
The first paragraph states what Poussin added to Guercino’s painting that was classical: “the Arcadian river god Alpheus and by transforming the decaying masonry into a classical sarcophagus.” Since B is not on that list, B has to be incorrect.
The discussion of the inscription occurs later in the passage, and I know it isn’t clearly related to the passage discussion of the epitaph, which the passage relates to the tomb and the death’s-head. So I can eliminate C.
The passage treats the shepherds and the tomb as two related, but distinct aspects of the passage. The shepherds in particular are read in the passage as potentially reacting to the tomb and what is on it, but the opposite is not so clearly suggested. That means D has to be incorrect.
I know that the historical significance of Poussin’s second painting is stated in paragraph 3; it indicates Poussin’s “radical break with the medieval, moralizing tradition.” The most obvious way to weaken this claim is with an answer choice that suggests Poussin did not break with a medieval moralizing tradition.
The changes introduced to Poussin’s second Arcadia painting don’t have anything to do with Guercino. And while it is true that Poussin’s Arcadia paintings depart from Guercino’s, whether or not they both have classical elements has no effect on the idea that Poussin broke with a medieval moralizing tradition. This is because the passage does not clearly suggest that Poussin’s classical elements had anything to do with his break from the medieval tradition, since those classical elements are mentioned in paragraph 1, and the break is mentioned in paragraph 3. Since I can’t see how A clearly weakens the passage argument about Poussin’s second Arcadia painting, I can eliminate A.
B has a similar problem to A: the right answer has to focus on Poussin, not Guercino. That’s enough for me to eliminate B.
This seems like an attractive answer choice, because C might suggest that we might not be able to discern Poussin’s perspective if one of his students finished the painting, but C is way too ambiguous to definitively weaken the passage argument. C might be right if it said that the painting was completely painted by one of Poussin’s students, but since the student only “completed” the painting, it’s entirely possible that Poussin could have directed the student to make the changes the author claims indicates a break with the medieval tradition. Since C does not clearly weaken the passage, I can eliminate it.
D is the last answer standing, and so it better be correct. Luckily, D matches my prediction. If later paintings done by Poussin have moralistic themes, that could at least suggest that perhaps Poussin did not really break with a medieval moralistic tradition. And if Poussin did not break with that tradition, then perhaps the second Arcadia painting should be read as part of that medieval tradition. Note that D only weakens the argument mentioned in the question stem: only makes the conclusion less likely to follow the evidence for it. It does not disprove it because it still leaves open the possibility that the author is right. For example, it’s possible that Poussin’s later paintings could have had moral themes, but in a way that fully departed from the medieval moral paintings that the author is referring to.
The fact that this term is not given with a clear indication of where it occurs makes me think that this has to be in line with the author’s main idea, and so describes Poussin’s second Arcadia painting. And that turns out to be true; in paragraph 4, the author states that “Poussin’s Louvre picture no longer shows a dramatic encounter with Death but a contemplative absorption in the idea of morality. We are confronted with a change from thinly veiled moralism to undisguised elegiac sentiment.” I know that elegiac here is contrasted with “dramatic,” so the right answer has to refer to something calm. I also know that because of the way the sentences are written, “elegiac sentiment” has to be a restatement of “contemplative absorption in the idea of morality,” so I’m also looking for a synonym of “contemplative.”
A does not clearly refer to contemplation or calmness, so I can eliminate A.
B, however, refers to both. “Serene” is a synonym of calm (and so is the opposite of “dramatic”), and “reflective is synonymous with “contemplative.” That makes B correct.
C has the same problem as A: it doesn’t clearly refer to either calmness or contemplation, and so C must also be incorrect.
D also lacks either of the two meanings it needs to reflect, so it’s also wrong.
This is a question that requires me to pay close attention to the actual text of the passage. Euripides is mentioned in paragraph 1: “The inquisitive spirit of Euripides, when not (as in the Bacchae) interpreting the gods as profound psychological forces, was capable of presenting them as shady seducers or discredited figures of fun.” The way that sentence is phrased suggests that it is presenting a range of depictions, from positive to negative. Since “shady seducers” is clearly bad, then it must be good to interpret the gods as “profound psychological forces.” That has to be the right answer.
There it is; I can choose it and move on.
Euripides is only mentioned in that one sentence. Since B is not in that sentence, B can’t be correct.
C has the same problem as B.
I know that “figures of fun” is definitely not a “favorable” portrayal of the gods, since it is presented as similar to “shady seducers,” which is definitely a negative description. That makes D wrong.
If Socrates said that he would obey his god, then it would seem to suggest that Socrates believed in gods. But though Socrates is part of a paragraph describing some “flippancy” with regards to the gods, the passage author is pretty careful to indicate that Socrates was accused of a very specific crime: “not believing in the gods in whom the city believes.” For Socrates to say that he would obey his god does not tell me who that god is. That god could be an Olympian god (the gods of the city), or it might not be. I would need to know more about what god Socrates did believe in in order to properly assess the effect of the new information in the question stem on the part of the passage the question stem refers to.
It’s possible that Socrates’ statement could refute the claim, because it at least suggests that Socrates believed in a god. But since I don’t know which god Socrates called his own, I can’t be sure that the quote refutes the claim in the question stem. It all depends on whether or not Socrates believed in an Olympian god or not. Since I can’t be sure of A, I should eliminate it.
B has the same problem as A: I can’t determine what effect the quote from the question stem would have on the passage argument about Socrates without knowing more about what god Socrates claimed was his. So B has to be incorrect.
C is definitely incorrect. If Socrates was referring to an Olympian figure, then that would mean that Socrates worshipped one of the gods in whom the city believes.
D has to be right: both because it is the last choice standing, but also because it makes sense. If Socrates is not referring to an Olympian figure, then the quote serves as proof that the prosecutors were right in accusing Socrates of not worshipping their gods: the Olympian gods.
This question is very similar to the previous one, except instead of asking about the effect of new information on Socrates’ prosecutors, it is asking about how the information affects the passage claim about the Stoics. The Stoics are mentioned in paragraph 2, where the author argues that even the Stoics saw the gods as “merely symbolic […] even the Stoics, for all their belief in divine Providence, reinterpreted and accommodated many individual deities as merely allegorical explanations of natural phenomena.” As long as I have this part of the passage argument in mind, I should be able to find a correct answer.
A has to be correct. Since the situation in the question stem describes the Stoics interpreting an individual deity (Zeus) to explain a natural phenomenon (thunder), the situation would support the claim made in the quote from paragraph 2. I can therefore choose A and move on.
B is very similar to A, and is suggested by the start of the sentence in which the Stoics are mentioned: “Many people had come to regard them as merely symbolic.” However, the description of the Stoics in paragraph 2 doesn’t directly suggest that they saw all the Olympian gods as merely symbolic. The passage actually tries to distinguish the Stoics from the “many people” mentioned in paragraph 2 by stating that they believed “in divine Providence,” which is to say that they still believed in some kind of divinity. Since the information in the question stem is insufficient to suggest that the Stoics believed the gods were just symbols, B can’t be correct.
The author acknowledges that the Stoics believed in divine providence in paragraph 2, and does not walk that claim back at all. The rhetorical effect that the author is going for actually depends on the Stoics believing in divine Providence, since the author is trying to say that even believers in the gods tended to see the gods in ways that were not strictly literal. C therefore has to be incorrect.
D is nonsensical enough to be incorrect. If referring to the gods as originators of natural forces is a slight to those gods (since it opens the door to suggesting that the gods are not actually real, that they are just convenient explanation of natural forces that are difficult to understand), then the information in the question stem would strengthen the claim that the Stoics slighted the gods, not weaken it.
I know that the first three paragraphs are about the decline of religious belief in the Greek (Hellenistic) world, while the fourth paragraph walks that back. The right answer will therefore be in that fourth paragraph. The fourth paragraph states that rather than declining, the Greek religion “had deviated, and continued to deviate throughout the Hellenistic age, from the traditional mainstream of the classical Olympian cults. They continued, it is true, to receive impressive ceremonial worship, but a person of this epoch no longer pinned his or her faith on these gods, but on a number of Divine Saviours.” So I know the right answer has to involve these Divine Saviours.
The comments from St. Paul come in paragraph 3, where they are quoted to suggest that the Greeks were becoming a godless people: “a ‘world without hope – and without God.’” Since St. Paul’s comments actually therefore suggest that religion might not have been vital in the Hellenistic world, A has to be incorrect.
B matches my prediction regarding the Divine Saviours, so I’m pretty sure it is correct. Just to be sure, though, I should double check the other answer choices to make sure they don’t refer to the Divine Saviours as well.
These two figures are mentioned in paragraph 3, when the author is still arguing that the Greeks were beginning to distance themselves from religion, so C can’t be correct.
D is also mentioned in paragraph 3 as a further degradation of the gods, and therefore the religion of the Greeks. There is therefore no way that D could serve as proof that religion was important to the Hellenistic world.
I know that the Iliad and the Homeric Hymns are mentioned in paragraph 1: where the author just begins to argue that the Greeks were departing from being “a religious people.” The specific reference that includes those two books is: “there had always been a tendency, at the same time, to treat the gods with a certain familiar flippancy—this is already very apparent in the Iliad and the Homeric Hymns.” So I know I’m looking for an answer that refers to how these two books may have treated the gods in an irreverent or playful way.
This is stated in the sentence following the mentions of the Iliad and the Homeric Hymns, which I know supports the same point that the mentions of the Iliad and the Homeric Hymns supports. I know this because that sentence ends by saying that these movements subjected the gods “to a further battering.” Since A and the Iliad and the Homeric Hymns are both evidence for the same claim, there’s no way that A could be correct.
B goes a bit further than the passage does. To say that the gods “were no longer a matter of serious concern” is to suggest that people stopped paying attention to the gods. All that the Iliad and the Homeric Hymns suggest is that even as the Greeks were religious, they also could make fun of the gods from time to time. Since B goes further than the passage does, it has to be incorrect.
C is almost a direct quote of the first half of the sentence that references the Iliad and the Homeric Hymns, so C has to be correct.
This is a reference to Socrates, at the end of the paragraph. Socrates is, like B, mentioned to support the same point that the Iliad and the Homeric Hymns reference, so those texts cannot be evidence for D.
Callimachus and Theocritus are mentioned in paragraph 2, where the author is arguing that the Greeks began to distance themselves from the gods. But since this question is asking about a piece of evidence for that claim, it makes sense to reread the section of the passage that mentions them: “Like Hellenistic sculptors, who began to represent some of these gods in much less idealistic forms than those their predecessors had favoured, the poets Callimachus and Theocritus showed that they were living in an age when the old gods were no longer a matter of belief or serious concern.” That implies that before Callimachus and Theocritus, poets treated the gods as a matter of belief and serious concern.
The only similarity to sculpture mentioned is how Callimachus and Theocritus treated the gods in a similar manner to sculptors of the time. But since the question is asking me about poets before Callimachus and Theocritus, A has to be incorrect.
B has a similar problem to A. Callimachus and Theocritus are cited as evidence that people from the early Hellenistic age started to insult the gods. So B describes Callimachus and Theocritus, and not the poets that preceded them.
I can eliminate C because it’s even stronger than the argument in paragraph 2 is.
D matches my prediction—“highly idealized” counts as the opposite of treating gods as if they “were no longer a matter of belief or serious concern”—so I can choose it and move on.
I know that the passage begins by describing the agrarian myth in America, and why it is so beloved. The second half of the passage then explains how that myth became increasingly untrue by 1860. The author never says, though, that Americans gave up the romance of the agrarian myth; the first paragraph actually suggests that the myth is still held. So the right answer has to say something about how this beloved myth became increasingly incongruous with actual agrarian life.
While the passage argues that the agrarian myth became increasingly dissonant with the actual experience of independent farmers, the author does not go so far as to suggest that the myth has no factual basis. It certainly did have some basis in the past, and even when the author finishes the thrust of the passage argument in paragraph 6, the author acknowledges that some parts of the myth still have a factual basis in reality, such as the fact that farmers “were a hardworking lot.” Since A goes too far, it has to be incorrect.
B, though, works as a correct answer. To say that the myth is a “sentimental representation” is to suggest that it is inaccurate: that it is over-romanticized in a way that distances the myth from reality. B is good enough for me to choose it and move on.
C is definitely incorrect, since the passage is about how the reality of farming began to depart from the myth.
D might be attractive because it superficially resembles B. But while the passage’s description of the agrarian myth and its relationship to the reality of farming does suggest that the myth is idealized, the passage primarily focuses on how actual farming began to become more commercial than the myth described: not that farming was necessarily worse or harder than the myth described. That makes D incorrect.
This is a straightforward passage check question, since the answer is the idea of paragraph 3: “In origin the agrarian myth was not a popular but a literary idea.” So the right answer has to mention literature.
A does, and so I can choose it and move on.
I can eliminate B because the passage does not refer to European estates. It only refers to “plantations or country estates” in America (paragraph 2).
Paragraph 3 says the opposite: that while the myth may have involved those farmers, it actually was “a preoccupation of the upper classes, of those who enjoyed a classical education, read pastoral poetry,” who owned “plantations or country estates.” Since none of that is going to describe yeoman farmers on small farms, C has to be wrong.
This is meant to be an attractive answer choice, since it generally resembles the argument in paragraph 3. It is true that the “virtues of the simple life of the farmer” were “romanticized” by an “elite.” But D is wrong because it distorts the passage. For the people who originated the agrarian myth to have “experimented with breeding stock, and owned plantations or country estates” is to suggest that this elite is rural, not urban.
The question stem does not give me enough to predict what the right answer will look like, so I will just need to evaluate the options by testing them against paragraph 2. I need to determine which options must be true for the claims about the agrarian myth to make sense.I. yeoman farmer are wholesome and honest. I mentions farmers who are “wholesome and honest,” which resembles the claim from the end of paragraph 2: that the yeoman farmer who lived the agrarian myth “was believed to have a wholesomeness and integrity impossible for the depraved populations of cities.” “Integrity” here can definitely mean being honest, so I is correct.
II. yeoman farmers are morally superior to most citizens. That quote from paragraph 2 also seems to suggest something like II, since it is generally the case that people are more heavily concentrated in cities than they are in the country. But II could also be wrong without my knowing exactly how many people lived in the cities relative to the whole population—since it is making a claim about “most” citizens, or over 50% of citizens—so I should be worried about choosing II.
III. agriculture deserves special treatment from the government. III, though, is also almost a direct quote from paragraph 2, which states that “agriculture, as a calling uniquely productive and uniquely important to society, had a special right to the concern and protection of government.” So I know that the right answer has to at least mention I and III.
A has to be incorrect, because III is also correct.
B also lacks III.
C lacks I, which I also know is correct.
D includes II, which I wasn’t feeling great about, but it is the last answer choice, and is the only one which includes I and III. So even if I have problems with the way this question is written, I can see how II could be right, and am strategically forced to choose D anyway because no other answer choice contains the options I know to be correct.
This is a direct reference to paragraph 4: “By the early nineteenth century it had become a mass creed, a part of the country’s political folklore and its nationalist ideology.” This is because the American Revolutionary War appeared “to many Americans as the victory of a band of embattled farmers over an empire” and so “seemed to confirm the moral and civic superiority of the yeoman.” The right answer thus has to mention something about the Revolutionary War.
Paragraph 4 doesn’t mention anything about some absence of patriotism that the agrarian myth filled, so I can eliminate A.
This resembles the quote from the passage, and includes the reference to the Revolutionary War that I was looking for. I can choose B confidently.
My work on the last question tells me that this isn’t the case. I also know that this is simply never stated as a reason why the myth became part of “a mass creed.” So C is wrong.
The author mentions D in paragraph 5, but this correspondence is not what the author claims made the myth so popular. So D has to be wrong.
This question again references paragraph 2, where the agrarian myth is mentioned: “The yeoman, who owned a small farm and worked it with the aid of his family, was the incarnation of the simple, honest, independent, healthy, happy human being.” I also know that the myth is tied to agriculture, “a calling uniquely productive and uniquely important to society, had a special right to the concern and protection of government.” The right answer has to mention at least some of those elements.
The entrepreneurial aspect of American farming is mentioned much later in the passage, and it is a reference to the reality of agrarian life in America, as opposed to the myth. So A has to be wrong.
B is a reference to paragraph 3, which explains that the agrarian myth began with a classically educated, wealthy class. The author makes this point to emphasize that the myth was not started by those that the myth describes, so B has to be incorrect.
Patriotism is mentioned later on in the passage as one of the effects of the myth, but nothing in paragraph 2 states that the yeoman farmers described by the agrarian myth were patriotic.
D is the last answer standing, so it has to be correct, and it clearly is. Paragraph 2 says that the yeoman farmer was a “healthy, happy human being,” and one engaged in a productive enterprise, so D has to be correct.
This is a straightforward main point question, so I can best prepare for it by keeping the main idea in mind: that the agrarian myth was a romanticized version of life that grew increasingly incongruous with the realities of farming. The easiest way to weaken this claim would be to have an answer choice with claims that life in nineteenth century America still resembled much of the agrarian myth, but it’s also important to remember that the right answer could affect any part of the passage argument, so the right answer might not resemble that prediction.
A actually helps to support the passage argument. To say that the agrarian myth is romanticized is to suggest that reality does not resemble the myth. A gives another way that the myth did not live up to reality, and so does not weaken the passage argument.
B also seems in line with the passage argument. The author spends most of the passage arguing that the agrarian myth was false, and that it was beneficial insofar as it helped encourage American patriotism. Since B follows the passage’s logic, it can’t weaken the passage.
C doesn’t match my prediction, but it does attack a central part of the passage argument: that the myth was widespread enough to affect American identity as much as it does. If only a very few people were ever affected by the myth, then that would definitely weaken the passage argument from paragraph 3: that the myth was “almost universally accepted in America during the last half of the eighteenth century.” That’s enough of an attack on the passage to make C correct.
Like A and B, D corresponds with the spirit of the passage argument, which is focused on explaining how the agrarian myth was a romanticized version of agrarian life in America. It also is in line with the argument from paragraph 6, which outlines how nineteenth century America really began to depart from the agrarian myth. D is too much in line with the passage argument to be correct.
This is a straightforward weakener, so I just need to make sure I know the main argument of the passage. The passage talks about a situation: the increasing energy constraints faced by the U.S. The passage then outlines two approaches to addressing that problem—the hard path and the soft path—in order to argue that while the soft path is gaining popularity, it also faces problems. So the right answer is likely to contradict either the idea that the soft path has become more popular, or that it is facing problems. I also know that the 3 wrong answers have to be consistent with information in the passage: that is, have to be stated or implied by the passage.
A is actually consistent with passage information. It would certainly explain why people are talking about the two paths, and why the soft path might be gaining popularity. A therefore has to be incorrect.
B is also consistent with passage information: specifically with paragraph 1, which ends with a list of pieces of legislation that environmentalists had helped pass. That makes B incorrect.
C is stated at the end of paragraph 2: “The turning point came in 1970, when U.S. oil production reached its peak and then began to decline.” So C is consistent with passage information.
D is left, and it matches my prediction, since it suggests that the soft energy path might not have the problems that the passage describes. Paragraph 4 mentions how the U.S. government has hurt the soft path: “A more rapid spread of this approach is being hindered by government (taxpayer) subsidies of the hard-path approach.” Since D contradicts this, D is correct.
And here we have a nice straightforward strengthener. Because the author outlines all the opposition that the soft-energy path has faced, and since the author does not provide any support at all for the hard-energy path, I can safely assume that the author would support the soft-energy path. That should be enough for me to find an answer.
It’s unclear how the author would feel about this. It’s possible that such acquisitions might help solar technology companies find the capital necessary to mature renewable energy technology, but it’s also possible that such acquisitions could be intended to hold these companies back. Such is what the author describes in the last paragraph, with oil companies buying up copper mines because of how important copper is to solar energy production. Since A is not clearly a strengthener, I can safely eliminate it.
The use of low-sulfur oil is mentioned in the passage as part of the reason why the U.S. finds itself in an energy crisis, but oil is also part of the hard path approach, which I predicted the author would argue against, so I can eliminate B.
Nuclear power is also listed as part of the hard path in paragraph 3, so I can eliminate C.
D works as the right answer, since government support would definitely help increase soft path energy in the country. It would at least more clearly be in favor of the soft path than A is, so I can feel good about choosing D.
I know that soft energy is most directly outlined in paragraph 3: “Soft technologies, on the other hand, use to the greatest possible extent nondepletable resources like sun, wind, and vegetation. They emphasize diversification and dispersal of energy sources so as to avoid in the future the sort of dependence we now have on fossil fuels.” That will help me go through the options.I. Tax credits for corporations that install solar panels in office buildings I would definitely help spread one of the soft technologies—solar technology—so I has to be correct.
II. A mandate to increase the ration of soft- to hard-energy sources by a specified amount within ten years Even though I don’t know how much of an increase II is talking about, any increase would likely be better than no increase in soft energy sources, so II has to be correct.
III. Cash incentives to homeowners who convert their heating systems from oil to natural gas Natural gas is mentioned as a hard-energy path source, so an advocate of the soft path wouldn’t clearly advocate III. That makes III incorrect.
I is a good option, but so is II.
B has both I and II, so I can choose it and move on.
III is incorrect, so C is wrong.
D has the same problem, so D is wrong.
Here’s another broad answer choice. With no clear indication of where I might need to look in the passage, I have to evaluate each answer choice and test whether or not it has a basis in the passage.
There is no basis for this in the passage, mainly because the passage only mentions how the U.S. opposes solar technologies. It is worth noting that A could very well be true, but without something in the passage to base this claim on, A has to be incorrect.
B has to be the correct answer. I know this because in paragraph 2, the author explains that environmental concerns “about air pollution led to fuel switching, especially by electric utilities, away from domestically produced coal to low-sulfur oil, which had to be imported.” This sentence would only make sense if the imported oil produced less pollution than coal.
The passage mentions oil company control over copper mines in the last paragraph, but all the passage claims is that “oil companies now control almost 60% of domestic copper production in the U.S.” Since the passage does not explain how much of global copper production these companies control, C can’t be correct.
I also can’t justify D, since natural gas is listed as part of the hard path: one of the ways the U.S. has reacted to declines in oil production. The opposition to the soft path and the subsidies for the hard path suggest that there has likely been an increase in natural gas consumption since the 1970s rather than a decrease. That makes D wrong.
To answer this question, I need to make sure I know what factors make the soft-energy path attractive. Those factors are implied by the second paragraph, where the author lists the reasons why the American energy market needs to decide between a soft and hard energy path: “the consumption pattern” and “prices” of oil. So one of these is likely to be the right answer. I can also figure out what the right answer should look like by considering what would make the hard-energy path unattractive. Since the hard energy path involves “the rapid expansion” of coal, oil, gas, and nuclear power (paragraph 3), the right answer should also suggest that there are downsides to pursuing such an approach.
A matches both my predictions: both because it is a factor in why power companies need to choose a path, and because it mentions a rise in the price of oil. Such a rise implies a decrease in supply, which in turn suggests that the hard-energy path, which is pursuing more sources of oil, wouldn’t work: thus making the soft-path more attractive. A therefore has to be right.
B has to be wrong because it makes nuclear power more attractive, and nuclear power is listed as part of a hard-energy path.
C is wrong for the same reason B is: because it too makes the hard-energy path more attractive, which in turn makes a soft-energy path less attractive.
D also supports a hard-energy path, since it gives a reason to keep producing oil: because it has taken care of imported oil, which the passage claims is a major driver of the oil cost increases and supply decreases which have forced the energy companies to choose a path. If the hard-energy path can keep those oil prices down, then that makes a soft-energy approach even less attractive.
The situation in the question stem describes a piece of evidence in favor of the soft-energy path. The second-half of the passage largely describes the hard-path and soft-path approach to energy, and barriers against the soft-path approach in the U.S., with the caveat being that hard-path companies are preemptively diversifying into soft-path approaches. I expect that the right answer will be something about how this inventor will be met with opposition generally, or with tacit support from a hard-path company.
Choice A reflects the passage’s caveat, regarding barriers against the hard-path approach, that utility companies will secretly diversify into soft-path approaches. A is a plausible choice, so I will keep it for now.
B is similar to A, insofar as both imply that this new technology will be met with success. But that seems to run against the passage argument about how much trouble soft energy has faced in the U.S. by the government, so B is also wrong.
The passage offers no evidence that hard path energy corporations would ever support soft path technology, so I can eliminate C.
Although the passage states that hard-path companies disparaged new forms of soft-path energy (i.e. solar, in the last paragraph), they actively diversify themselves into those new forms. Therefore, the passage does not imply that the nuclear power industry will punish customers of soft-path industries, and D is most likely incorrect, leaving me with A.
This question is asking me about how a part of the passage is presented: specifically if and how it is supported. To answer a question like this, I need to check the reference and what occurs around it in paragraph 6: “To be unaware that a technology comes equipped with a program for social change, to maintain that technology is neutral, to make the assumption that technology is always a friend to culture is, at this late hour, stupidly plain and simple. . . . Introduce the alphabet to a culture and you change its cognitive habits, its social relations, its notions of community, history, and religion. Introduce the printing press with movable type and you do the same.” The last sentence suggests that both the printing press and the alphabet are examples of technologies that come “equipped with a program for social change.” Since the assertion is itself evidence for a claim, it is not supported by the passage: something that the right answer will have to mention.
The passage doesn’t mention low literacy rates in the U.S., so I can safely and quickly eliminate A.
I don’t see any objective data about the effects of introducing an alphabet in the passage. There are no surveys, or data points, so B has to be wrong.
C matches my prediction, since the claim about the alphabet is not explicitly supported by passage information. And since the author does not contradict the claim, it could perhaps be true. C therefore works as the right answer.
There is no such contradiction here, even if the passage is about television watching. The passage actually suggests that television and the alphabet are the same insofar as both are technologies that changed cultures when they emerged. That makes D incorrect.
This is a weakener for the author’s main argument: that the U.S. is spiraling towards Huxley’s dystopia, which describes as one in which “culture becomes a burlesque” (paragraph 1). Paragraph 3 puts it more precisely: “For [the U.S.] is engaged in the world’s most ambitious experiment to accommodate itself to the technological distractions made possible by the electric plug.” This is a problem because, as the author points out in the last paragraph, Huxley “believed . . . that we are in a race between education and disaster, and he wrote continuously about the necessity of our understanding the politics and epistemology of media. For in the end, he was trying to tell us that what afflicted the people in Brave New World was not that they were laughing instead of thinking but that they did not know what they were laughing about and why they had stopped thinking.” So if the problem in Huxley’s dystopia is that television transforms a culture into one that no longer thinks about politics, it would make sense for the right answer to either suggest that people who watch television do think about politics, or that television somehow encourages the critical thinking that Huxley worried was disappearing.
A matches one of my predictions: that watching television would not necessarily keep people from thinking about politics. I’m pretty confident about A, but this is a complex enough passage that it makes sense to eliminate the other choices just to be sure.
I can eliminate B because it would definitely strengthen the passage argument, since it describes television watchers who stop thinking about politics.
C runs into the same problem that B does: it describes people who are into television watching (that’s how they would know the name of the judge on “The People’s Court”) who stop thinking about politics (which is why they can’t name the U.S. chief justice). C has to be incorrect.
I don’t see why D would weaken the passage argument. Even if it were the case that more people were familiar with Orwell than Huxley, that would not necessarily suggest that society would grow less Huxleyan. Ultimately, the point about how Huxleyan America is becoming has nothing to do with how many people know who Huxley is. That makes D incorrect.
This question is asking me to apply the passage logic to a specific situation. What stands out about the passage discussion of Huxley’s predicted future is how invisible it is. This is stated in paragraph 4, which describes those who try to warn about the Huxleyan future as having to “often raise their voices to a near-hysterical pitch, inviting the charge that they are everything from wimps to public nuisances to Jeremiahs.” That suggests that if a news commentator tried to warn about the future Huxley feared, that news commentator would be ridiculed.
Nothing in the passage suggests that viewers would listen to the news commentator. There isn’t really enough hope in the passage for A to be correct.
B is effectively the same answer as A, insofar as both suggest that people will try to keep the Huxleyan future from happening. And since nothing in the passage suggests that, B has to be incorrect.
C matches my prediction, and the quote from the passage. It makes sense to call someone who is “irrational or needlessly alarming viewers” a “public nuisance.” C is therefore correct.
D is again similar to A and B in suggesting that people would try to stop the Huxleyan future from happening, when it is people’s inability to recognize that a dystopia is coming that makes the Huxleyan future so dangerous. D is therefore incorrect.
The challenge with this question is figuring out what part of the passage can speak to the situation in the question stem. Triviality is not very explicitly discussed in the passage, but politics definitely is. I know from the last paragraph that the author is worried that people stop thinking about politics because of television. So I’m probably looking for an answer choice which implies that the public is not paying attention.
This doesn’t seem to mention anything about people not thinking, since even voting for the most trivial candidate would require enough thought to identify who the most trivial candidate is. So I can eliminate A.
I can eliminate B for the same reason: because voting for the least trivial candidate would require the political awareness that the author fears is already lacking in America.
C also describes political engagement that the passage author fears is no longer possible in America, so it has to be wrong.
D matches my prediction, since it describes the public not thinking enough about those in the election to notice when the candidates and the discussion are trivial: are not what one would want out of political discourse. D is therefore the correct answer.
This claim is made at the end of paragraph 3, but it is not fully explained until paragraph 6, where the author discusses the effects of technology: “To be unaware that a technology comes equipped with a program for social change, to maintain that technology is neutral, to make the assumption that technology is always a friend to culture is, at this late hour, stupidity plain and simple.” In other words, television is relevant because it changes society in ways that the author is clearly worried about.I. television discusses vital matters. I can’t be correct because the opposite is true. The author says that the U.S. is headed towards a Huxleyan future because it is trying “to accommodate itself to the technological distractions made possible by the electric plug.” To call what television puts out “distractions” is to say that television does not discuss vital matters.
II. television is changing people’s way of thinking. II has to be correct, because it is in line with paragraph 6: the idea that technology like the television changes how people think. This is clearly paralleled in that paragraph’s description of the alphabet, which the author argues could change a culture’s “cognitive habits.”
III. technology can cause negative social changes. III too must be correct, since the author is clearly worried about the negative social effects of focusing on television, such as people’s disengagement from politics.
I is incorrect.
III is also correct.
I here makes C wrong.
D is the only answer left and it matches my prediction: that II and III are correct.
This is a reference to the “political message” of the term “woman writer,” which is given in paragraph 5. There, the author imagines that the term “woman writer” means separating themselves off with that designation from the community of writers. The author then imagines that the term says, “we will separate ourselves only temporarily, during this strengthening period, and then, when we can rejoin the world with power and dignity in our hands, we will rejoin it and declare ourselves for the unity of the human species.” So women writers will end their voluntary segregation when they have “power and dignity,” after a “strengthening period.” That should be enough for me to find the right answer.
A looks good because it sounds like the hopeful end result of going through a “strengthening period.” Paragraph 5 also has the term “woman writer” saying that it wants women “to become politically strong,” so A has to be right.
The paragraph states that the message of “woman writer” says “of course we believe in humanity as a whole.” It does not ever suggest that literature does not recognize humanity-as-a-whole: that this is a problem that women writers will address. The author actually argues in the last paragraph that women writers will make literature less holistic: less able to recognize humanity as a whole. So B has to be wrong.
Imagination is never mentioned as a factor in the retaking of the term writer by women writers in paragraph 5. This is because the author seems to believe that the term “woman writer” is a political term (paragraph 1). So C is incorrect.
A new generation of women writers are brought up in the last paragraph, where the author mentions that this generation will be negatively affected by women writers. Since D is not a factor in the return of women writers, D has to be incorrect.
I know this claim comes from paragraph 4: “In art, feminism is that idea which opposes segregation; which means to abolish mythological division; which declares that the imagination cannot be ‘set’ free, because it is already free.” Given that the author is so focused on how much the term women writers divides, I’m therefore probably looking for an answer choice which brings up how feminism unites.
This is described as one of the effects of the term woman writer in paragraph 5, in which the term allows women writers to “for a little while gather together.” The author would likely not disagree that women writers deserve a common ground, but the author would object to the possibility that A is limited to women writers, since the author argues that feminism unites everyone and argues against divisions, including divisions of gender.
B is a direct quote from the explanation of feminism as a force in paragraph 4, so it has to be correct.
The author argues the opposite: that feminism should not divide. So I can eliminate C.
For D to be correct, the passage would have to imply that women writers do not accept humanity-as-a-whole. The author never implies this in paragraph 4, so D has to be wrong.
To answer this question, it helps to remind myself that while the question stem indicates that I’m looking for an answer choice that the author would disagree with, that the wrong answers have to be choices that the author would agree with.
A has to be the correct answer. I know the author strongly argues against this idea because the author accuses women writers of encouraging this mindset, and the author is primarily arguing against the term women writers. This is very clear by paragraph 4: “Does a ‘woman writer’ have a body of separate experience by virtue of being a woman? It was this myth-fed condition of segregation that classical feminism was created to bring to an end.” Since the author calls A the result of a “myth-fed condition,” there is no way that the author would agree with A.
B twists the logic of the passage. While it is true that the author dislikes the term women writers because it is a political term rather than an artistic one, and while the author makes it clear that writers are not politicians (paragraph 5), the author is for freedom of expression in general, and so would not clearly oppose women who wanted to express their political viewpoints. B is therefore incorrect.
The author would likely agree with C, since it is stated in paragraph 3: “There is a human component to literature that does not separate writers by sex but that – on the contrary – engenders sympathies from sex to sex.”
If women earned less (or more) than men, that would be evidence of segregation. The author explicitly argues against segregation of any kind (paragraph 4), so D has to be wrong.
The “eventually” here tells me that I need to look at the last paragraph: where the author lays out the potential consequences of using the term “women writers.” There, the author states “Writers will very soon find themselves born into one of two categories, woman writer or writer, and all the writers will be expected to be male.” So the answer will likely either mention the breaking down of writers into two categories, or the idea that writers will all be expected to be male.
The author seems to imply that those who use the term “woman writer” would be opposed to classical feminism in paragraph 4. Classical feminists want to unite, while woman writers end up dividing. A therefore has to be wrong.
B is incorrect because the author does not claim that the women will become political. I know from paragraph 1 that the author believes that to take on the term “woman writer” is to become a primarily political being; that doesn’t happen in the future.
This is not a claim made in the last paragraph, so I can eliminate it.
This, however, is almost an exact quote from the end of the passage, so I can choose it and move on.
I’m looking for an answer choice which contains something from the passage that would conflict with the situation in the question stem. The author liking a work written by a woman writer would be surprising because the author sees women writers as political, and “the language of politics is not writers’ language” (paragraph 2). In other words, political writing should make for bad writing, or at least writing that the author would dislike. So I’m probably looking for an answer choice which mentions the political orientation of works by women writers.
I can eliminate A because the situation above doesn’t clearly attack A, or is in conflict with A. Classical feminism isn’t clearly a part of the process of the author liking or disliking literature.
I also don’t see how B could be correct. B is about women writers and what they need or benefit from, but nothing in the question stem comments on that. It is instead focused on the writing by women, and what the author wants or needs.
C is definitely wrong because it mentions men, and the information in the question stem is focused on works by women writers. That means C doesn’t clearly relate to the passage argument.
D is the only answer left and it clearly matches my prediction by mentioning how politics affects the writing of women writers, and so must be correct.
To answer this question, I need to find where the passage defines literature, either implicitly or explicitly. One reference is paragraph 3: “There is a human component to literature that does not separate writers by sex but that – on the contrary – engenders sympathies from sex to sex, from condition to condition, from experience to experience, from like to like, and from unlike to unlike. Literature universalizes.” So I’m probably looking for an answer choice which claims that literature universalizes: helps people to understand each other.
A is pretty on the nose, insofar as it directly references the claim that literature “engenders sympathies from sex to sex.” That’s close enough to “universalizing” that A has to be right.
This is a political subject, and a form that is often used in political writing. The author explicitly says that “the language of politics is not writers’ language. Politics begins with premises; imagination goes in search of them.” Since B would likely use the language of politics, the author is likely to claim that it is not literature. That makes B incorrect.
C attacks one group of people on behalf of another. That goes against the idea that “Literature universalizes,” so I can safely eliminate C.
D is, like B, explicitly political, and so would use an approach that the author would find non-literary. I can therefore eliminate D.
The quote is a description of the Generalized School Readiness Program. My initial reading of the passage makes it clear that the author is pretty unhappy with that program, so I know the author is going to have a negative evaluation of the theory of education described in the question stem. The specific quote from paragraph 4 gives me more data on how the author feels about the Generalized School Readiness Program: “They are merely following the usual practice at the professional level of education of treating learning as an abstraction that has little to do with the learner.” So I know that this theory of education is common enough to be “usual,” but also that the author dislikes it. That should be enough to find a right answer.
The author clearly disagrees with the program mentioned in the question stem, but I’m not sure that the program is either practical or ethical based on the quote from paragraph 4, so I can safely eliminate A.
I know that the theory of education described in the question stem is “the usual practice,” so it has to be generally accepted. That means I can eliminate B.
C matches my prediction: that these theories are generally accepted (“the usual practice at the professional level of education”), while the rest of the passage goes on about how ineffective the program is. C is therefore the right answer.
Again, I don’t know how practical the program quoted in the question stem is, so I can’t pick D.
A quick scan of the passage points me to the first sentence of 7: “In the literature promoting their approach, the advocates of generalized readiness are clearly directing their appeal to school administrators.” So the right answer will likely say something to that effect.
Consumers aren’t mentioned in that claim from paragraph 7, so A is incorrect.
There’s no clear accusation of fraud here. The passage does bring up B as a possibility for why people support this program at the end of paragraph 3, but the author only speculates about this, and does not actually claim that this is the motivation for the Generalized School Readiness program. So B has to be incorrect.
C is also not mentioned in that sentence from paragraph 7, so I can eliminate C.
D, however, is mentioned in the quote which states that the supporters of this program are “directing their appeal to school administrators,” so D has to be correct.
I know that developmentally appropriate instruction is mentioned in the last paragraph because it is an italicized term, and so stands out: “Developmentally appropriate instruction, which parents are likely to judge the more reasonable approach, appears to be hard to sell to decision makers concerned with uniformity.” So the answer will have something to do with administrators wanting instruction to be uniform, or the same.
The quote does say that parents support developmentally appropriate instruction, but the passage does not say that parents do not understand learning. Since A goes well beyond what the passage does, it has to be incorrect.
Nothing in the passage suggests that developmentally appropriate instruction is untested, so I can eliminate B.
I can’t tell what traditional methods are, so I don’t know if developmentally appropriate instruction is a departure from tradition. However, I do know that developmentally appropriate instruction is opposed to uniformity, which means that it is individualized. That means I have some reason to choose C, even if I’m not sure about the entire answer choice. The other answers so far have been clearly wrong, so my best strategy here is to just double check D to see if it is also clearly wrong.
“Slick presentations” are mentioned earlier in the paragraph, before the author mentions developmentally appropriate instruction. So D actually describes Generalized School Readiness, and not developmentally appropriate instruction. D therefore can’t be a reason why administrators are reluctant to institute developmentally appropriate instruction, which makes D wrong and C right.
This is an analogy question about different approaches, so I should expect this question to be difficult. I know that the author thinks that the Generalized School Readiness Program is bad, while the author is lightly implied to be good in the last paragraph. But I probably need a more detailed sense of both to find an answer choice that resembles them. The author’s major contentions about the Generalized School Readiness program is that it is not sensitive to students; those who use the program treat “learning as an abstraction that has little to do with the learner” (paragraph 4). Developmentally appropriate teaching, on the other hand, is somehow a problem because it disrupts “uniformity” (last paragraph), and so likely treats different students differently. So I’m looking for an answer choice which mentions, first, a rigid and insensitive system, and then mentions second one that treats different students differently most likely because that approach is more sensitive to how individual students are actually learning.
A reverses the terms. Hand plowing is more individualized, and so should correspond with developmentally appropriate teaching, while a mechanized cultivator is enough of a cookie-cutter approach that it should resemble the Generalized Readiness program. The right answer to a question like this will parallel the parts of the passage mentioned in the question stem perfectly; since this does not, A has to be wrong.
B, though, is a great answer. The Generalized Readiness program is a cookie-cutter approach, as is sending a yearly form letter: a letter that is reused multiple times for different people. And sending personal notes to one’s friends will not produce uniform letters, since each friend should receive something different; that resembles developmentally appropriate teaching. B is therefore correct.
C is wrong because the only difference between the two approaches to fishing is how many fish they affect, while the educational approaches mentioned in the question stem differ in terms of the quality of attention individual students get. That’s too much dissimilarity between C and the passage to make C correct.
D is meant to be the tempting answer choice, since we tend to imagine that family meals cooked at home are more individualized than food we get at restaurants. But D has to be wrong because, unlike B, it does not identify a process whereby different people get different things. Every member of a family will receive the same meal when the meal is cooked at home. Since D lacks this basic similarity to developmentally appropriate teaching, D has to be wrong.
Reliability and validity are mentioned in paragraph 2, where the author explains that it is important to measure problems in education: “The appropriate instrument for measuring educational effectiveness is a test noted for its reliability and validity.” So since the question stem describes such a test, the right answer has to be something about measuring educational effectiveness.
A fits, since high school curricula is synonymous with education, and the performance of a high school curricula is a measure of educational effectiveness. I would choose A and move on.
Honesty isn’t mentioned in paragraph 2, so I can eliminate B.
Testing principles aren’t mentioned in paragraph 2 either, so C is gone.
And future income is also not mentioned in paragraph 2, so D is incorrect.
As with most LEAST questions, it is easier to predict what the right answer looks like than what the wrong answer looks like. The passage goes on and on regarding all the useless cleaning that the Mediterranean fleet did, so I know that the main idea has something to do with that. The main idea is best crystallized in paragraph 3; to say that “This cult of brightwork originated in the need to keep the men busy” is also to say that it was not strictly necessary, and was in fact, in some instances, harmful. So I know that the wrong answers will be quotes from the passage which supports this idea, while the right answer could be random.
This is quoted in paragraph 2: “Massive armored watertight doors were taken off their hinges and filed and rubbed until they gleamed—and were no longer watertight.” Since A is an instance of cleaning that is not only unnecessary, but was potentially harmful to ships, A supports the passage argument, and so is incorrect.
This is a quote from the last paragraph, which begins by noting that “Gunnery could not be wholly avoided, as admiralty order decreed that target practice be held once every three months.” The idea that the ammo was just dropped overboard is indicative of how much people wanted to keep the ship clean—since paragraph 5 states that “firing the guns spread dirt and grime”—that was potentially harmful, insofar as sailors dumped ammo instead of actually practicing with it. B therefore also supports the passage argument, and so is incorrect.
C, however, does not clearly support the passage argument. Coal is dirty, and so it’s hard to see how coal loading competitions are in line with the passage description of brightwork. C is also one of the few points in the passage that comes from paragraph 1, which leads into the argument about brightwork, and so is not actually a part of it. Instead, paragraph 1 establishes that there was great competition between ship crews, which included coaling competitions; paragraph 2 states that another competition, polishing, was the fiercest. So since C leads into, but does not directly support, the main passage argument, C is correct.
This comes up in paragraph 4, when the author quotes Vice Admiral K.G.B. Dewar, who mentioned that the beauty of a ship hid the rats that infested it. The inaccessible rooms were rendered inaccessible to keep beautiful rooms looking beautiful, and so D definitely supports the idea that the culture of brightwork could hurt ships. D is therefore incorrect.
This is a main idea question that’s asking me to deduce how the passage argument would react to the situation in the question stem. Since the question is asking me what the ships would avoid using, and since I know the passage argument is all about how clean these ships were, I can imagine that the right answer must be that these ships would have avoided using whatever was dirtiest. I imagine the steam engines would have been dirtier than sails because the former uses a dirty fuel while the latter does not, so I think the right answer will have to be the steam engines because of how dirty they were.
The passage does not play up a sense of tradition behind a method of propulsion, so I can eliminate A.
B describes a problem with cleanliness, so I’m pretty sure that B has to be the right answer.
If anything, paragraph 3—where sails and steam engines are described—suggests the opposite: that sailors on ships with sails would have known how to maneuver sails, since managing the sails is one way that the sailors would be kept busy.
The passage does not discuss enthusiasm for new technology, so I can safely eliminate D.
I know that the passage argues that the focus on cleanliness seemed to hurt the navy, so potential reforms are likely going to attack the culture of brightwork, and anything which may encourage people to focus too much on cleanliness and not enough on effectiveness. In other words, this is secretly a weakener question.I. the establishment of strict gunnery practice. I makes sense, since the last two paragraphs argue that gunnery was so dirty that ships avoided firing their guns, and sought to dodge mandatory gunnery practice. The last paragraph states that even when sailors did gunnery practice, it was quite loose, so I can imagine that strict gunnery practice would force sailors to set aside their disdain for the grime of the guns in order to get better at using them.
II. an emphasis on practicality over appearance. II also makes sense, since one way to rephrase the passage argument is to say that it describes a navy that emphasizes appearance over practicality. For the navy to emphasize practicality over appearance would have to improve the navy’s preparedness.
III. the elimination of busywork chores and ceremony. III, though, is not clearly correct. I know from paragraph 3 that the passage blames the need to keep men busy as the root cause for this emphasis on cleanliness, so it kind of makes sense that eliminating busywork would attack this culture of brightness. Ceremony also feels like a thing of appearance, rather than preparedness. But what paragraph 3 actually states is that it is the lack of busywork—a lack caused by the switch to steam engines, which took away much of the work sailors would have normally had in managing the sails—which caused this cult of brightwork to emerge. So even though the polishing is a form of busywork, the lack of busywork is still implied to be bad for the navy’s preparedness, which means that I don’t have a good reason to pick III. So I’m looking for an answer choice that mentions I and II only.
II is right, but I need I as well.
III is wrong, just because I can’t easily justify it.
C matches my prediction, so it is correct.
And D is wrong because III is incorrect.
This is really just testing my grip on the main idea; the right answer will be a plausible explanation for what the passage depicts as a quirky and strange practice. Why would a fleet polish itself to the point of being less combat effective and seaworthy? I have to assume that they had a good reason: that there had to have been some value in having such beautiful ships.
A definitely suggests that there was some value in having beautiful ships, so it’s a tempting answer choice. However, there’s a chance that I’m just jumping at an answer choice this time, so I’ll feel better if I eliminate the others.
I don’t have any information that suggests that the queen demanded the navy act as the passage describes, so I can eliminate B.
The description of how ammo was used during this time doesn’t clearly suggest that money was an object. If C were true, then people wouldn’t have dumped ammo overboard. C has to be wrong.
This might be the most tempting answer choice, but while it might explain why the British navy didn’t worry so much about its combat effectiveness, it does not explain why it would choose to focus on polishing ships instead. D therefore isn’t a direct explanation for the practices described in the passage, so I can eliminate it.
This claim is made in paragraph 2 so I know it most broadly has to be an example of how absurd it was that people focused so much on polishing ships to make them look beautiful.
A matches that prediction. I can confirm this by looking at the quote; by saying that “On some ships, even the ring bolts on deck were polished and fitted with little flannel nightcaps,” and specifically by saying “even,” the author is suggesting that what was done to these ring bolts was extreme, or “absurd.” I would therefore choose A and move on.
The phrasing implies that there is something silly and extreme about what was done to the ring bolts, while B implies that what was done to the ring bolts is admirable in some way. So B has to be incorrect.
While the author does argue this elsewhere in the passage, the passage does not say what ring bolts are for, so it can’t be arguing that the specific way these ring bolts were treated got in the way of what they were meant to do.
And I can eliminate D because the paragraph does not suggest that treating ring bolts in this way was a navy standard; it was instead something that “some ships” did.
This is meant to be a paradox question: a question that presents two statements that seem to contradict each other, but that requires me to explain why they don’t actually contradict one another. The passage argues that some British ships were so focused on appearances that it got in the way of their military performance, while the information in the question stem states that just fifteen years later, British warships “functioned well,” which I take to mean that they were combat-worthy. So I’m looking for an answer choice which will indicate that there was some change in behavior: that something made people stop paying attention to appearances to the point of compromising the battle-worthiness of the British vessels.
The passage doesn’t actually state that crews were inadequately trained. The closest the passage gets is in suggesting that crews may have been out of practice with guns, but the passage never suggests that sailors polished the ships because they were not trained well. So A would not explain a shift in behavior, which makes it incorrect.
I don’t have any information in the passage about what battle conditions the navy was prepared for, so B doesn’t have enough of a basis in passage information to be correct.
C, though, matches my prediction insofar as it mentions a change in sailor behavior. The last two paragraphs argue that gunnery officers and their demands for more gunnery training were not being listened to because the guns were dirty when used. If gunnery officers were listened to, then it at least makes it possible that British ships could have been prepared for combat.
This is a hilarious answer choice—that the British were fighting enemies that were as unprepared as they were—but having an enemy as inept as you are is not enough to explain why you would function well. It just means that both you and your enemy functioned poorly. In other words, the right answer needs to talk about some change in the British navy, which D does not. So D must be wrong.
The question stem’s ending with “take into account” points me to the last paragraph: “The cultural and social context, the level of development of each country, are factors that have to be taken into account. What has especially to be considered is the general trend in a country: Has there been a movement toward greater human rights or away from them?”I. the country’s level of development. I is a straight up quote from that last paragraph, so it has to be right.
II. the country’s social and cultural circumstances. II sounds like “social context,” so it has to be right.
III. whether the country is moving toward or away from greater human rights. III is also mentioned in that quote, so it is correct.
A is missing I and III.
B is missing I and II.
C is missing III.
D is the only answer choice which has all three options, so it is the right answer.
D is the only answer choice which has all three options, so it is the right answer.
The paragraph definitely begins by talking about the origins of the concept of human rights, but the author shifts the passage argument a little after that to mention that these non-Western cultures had some familiarity with human rights before the West brought that concept to them. So since the first paragraph shifts away from A before mentioning non-Western cultures, A has to be incorrect.
This matches my prediction, and so I can choose it and move on.
The passage doesn’t mention any resistance at all, so I can eliminate C.
The passage actually admits the opposite: that it is possible that these cultures might have lacked “explicit covenants,” that is, explicit mentions of human rights. That makes D wrong.
This question is asking me to determine the function of a claim: what role that claim plays in the author’s argument. These are very prominent documents that I know are mentioned in the first paragraph, but what stands out to me is that they immediately follow the main claim of that paragraph: “Human rights as an idea, as an issue in religious, political, and moral philosophy, has an ancient and illustrious pedigree.” So that means that those documents are meant to illustrate this claim.
This happens later in the passage, as my work on the previous question shows, after the paragraph has shifted to a supporting argument, and so A is not why the author mentions those prominent documents in the question stem.
This is certainly implied by the passage argument, but the author isn’t trying to argue B. For that to be the case, the author would have to first claim that the English Bill of Rights was the only early document to promote human rights in order to argue against that claim. Since B is not what the author intended to argue, B has to be wrong.
This claim is made in the second paragraph, where the author argues a different point, and so definitely doesn’t have anything to do with the documents mentioned in the first.
D is the last answer choice, and it fits my prediction, since it just restates the first sentence of paragraph 1. D is therefore correct.
This question stem suggests that the passage provides some way to evaluate the situation described, so my first priority needs to be to figure out what part of the argument I should draw from. The only part of the passage that makes sense to me is the last paragraph, where the passage talks about countries trending towards or away from human rights. That paragraph implies that it is a good thing if a country moves closer to human rights, which the country in the question stem seems to do, since it moves from a repressive dictator to someone who does not want to persecute citizens. I don’t see much else there to draw on to help me figure out what the right answer will look like, so hopefully that will be enough for me to find the right answer.
A makes sense, and aligns with my prediction. The country in the question stem is moving away from human rights abuses, and since the author implies that is a good thing in the last paragraph, it makes sense that the country should be praised. I would choose A and move on.
I don’t see anything in that last paragraph which claims that there should be any worry about more human rights abuses, so I can eliminate B.
I don’t know anything about the economy of the question mentioned in the question stem, so I can eliminate C.
The author seems to be explicitly arguing against D in the last paragraph. Rather than blame a country for their past failings, the author instead chooses to emphasize “the general trend in a country”: how it is changing how, as opposed to how it was. That’s enough to make D incorrect.
The passage is focused on human rights, so I’m guessing that the author would want to see violations of human rights forbidden by international law. The author outlines what human rights does not allow in the last paragraph: “However underdeveloped a country, there is no convincing argument in favor of torture, or arbitrary execution, of keeping sections of a population or a whole population in a state of slavery.” So the right answer will be synonymous with one of these.
This doesn’t resemble anything on the list, so A is incorrect.
B also doesn’t resemble anything on the list from the last paragraph, so it has to be wrong.
C is also not there, so it is wrong.
D, though, is definitely there: “there is no convincing argument in favor of torture.” Since the lack of any argument for these practices is used as evidence for the next sentence of the last paragraph—“The case for human rights is unassailable”—D must be something that the passage author would want to see forbidden. D is therefore correct.
This is a straightforward weakener, but also a general one; it’s not targeting any specific argument. So while I should be ready to find an answer that weakens the main argument—“The case for human rights is unassailable”—I should also be prepared for the right answer to challenge any argument made in the passage.
I can eliminate A, because it seems to actually stand up for human rights. If citizens have the “right” to disobey unjust laws—to challenge the system by which citizens derive most of their rights—then that must be an argument in favor of human rights. Since the author is interested in arguing for human rights, A would strengthen, not weaken the passage argument.
B, however, could weaken the passage argument. Specifically, it could weaken paragraph 3, where the author is unhappy with “the failure of the United Nations to live up to early expectations and to become an effective instrument for the promotion of human rights.” This desire for the United Nations to help promote human rights is ultimately the hope that an organization can meddle with the internal affairs of countries that are violating human rights. If B is true, though, then the United Nations would be a lot less likely to be able to promote human rights. I can choose B and move on.
The last paragraph seems to allow for C, since the author there advocates taking a country’s context into account when evaluating its human rights record. So C has to be incorrect.
The passage never says that passing human rights laws will immediately stop all oppression, so D isn’t related enough to what the passage actually says to weaken it. D therefore has to be wrong.
Quantum mechanics is mentioned in paragraph 3, where the author brings up how “it would be a mistake to assume that world views always crumble in the face of new scientific research traditions which challenge them.” Quantum mechanics is then raised as an example of this idea, so I know that the right answer will have to mention that quantum mechanics is a theory that is not changing people’s worldview.
The passage argues the opposite: that people find quantum mechanics unintuitive, and so in spite of the science, “most people still conceive of the world as being populated by substantial objects, with fixed and precise properties.” So A can’t be right.
B is correct. The author brings up both quantum mechanics and behaviorism in paragraph 3 as examples of scientific theories that end up challenging people’s worldviews.
The passage never seems to directly suggest that quantum mechanics is incompatible with the notion of natural scientific laws. In fact, the last paragraph seems to suggest the opposite: that quantum mechanics and all other scientific theories are examples of our belief that the universe is governed by natural laws, such as the laws of quantum mechanics. That makes C incorrect.
D is definitely not the case, because while the passage talks about scientific research which does end up changing worldviews, quantum mechanics is not one of them.
This is a reference to the last paragraph: “Virtually all of our social institutions, most of our social and political theory, and the bulk of our moral philosophy are still based on a world view seemingly incompatible with a law-governed universe.” That world view is elaborated on in the previous paragraph and the last one. Paragraph 4 claims that “Ever since the seventeenth century, the dominant research traditions within the physical sciences have presupposed that all physical changes are subject to invariable natural laws.” But paragraph 5 claims that even though those laws have to apply to humans, “in our own time as much in the seventeenth century, very few people are prepared to abandon the conviction that human beings (and some of the higher animals) have a degree of indetermination in their actions in thoughts.” In other words, even though we believe in a universe that is bound by laws, we also believe in free will: in the idea that we humans live in a way in which our decisions are not ruled by laws. So that tells me that our social, political and moral beliefs, which are all inflected by the belief in free will, contradict the science. I know this has to be the right answer because that’s very much in keeping with the main idea of the passage: that sometimes, scientific theories fail to change dominant conflicting worldviews.
The reference to the last paragraph suggests the opposite: that our social, political and moral beliefs are all inflected by the belief that sometimes, laws don’t apply to everyone. So A has to be wrong.
The passage does not talk about social, political, and moral beliefs in relation to other cultural attitudes. The passage is instead contrasting culture and science. So B is out of the passage scope, which makes it incorrect.
C matches my prediction—that our beliefs are in conflict with the science—and so I can choose it and move on.
The passage mentions Darwinism and Marxism as moments when science came to change dominant worldviews. But that is not directly tied into our social, political and moral beliefs in the passage, so I can safely eliminate D.
Newton is mentioned in paragraph 2 as someone whose work “went violently counter to many of the most cherished beliefs of the age” but that also eventually created “a new world view which could be reconciled with the scientific research traditions.” In other words, Newton’s science conflicted with the way people thought of themselves and the world at the time, but ended up changing those perspectives. If Einstein’s theory of relativity resembles Newton’s theories, then I need an answer choice which suggests that Einstein’s theories of relativity was initially rejected, but then changed how people saw the world.
A does not match what happened to Newton since it doesn’t mention Einstein’s theory getting rejected, so A has to be wrong.
B, however, fits what happened to Newton, so B is correct.
The discussion of Newton does not include anyone ignoring the implications of Newtonian physics, so C has to be wrong.
D might fit the first half of Newton’s story, but it does not include the most important part of that story: how Newton’s theories eventually changed how people saw the world. D is therefore less complete than B, which makes D incorrect and B correct.
This claim is made in the last paragraph. It is an entailment of an example that starts in the prior paragraph: that “all physical changes are subject to invariable natural laws.” So that has to be the answer.
This doesn’t resemble my prediction. It also doesn’t resemble the claim mentioned in the question stem, since it does not talk about natural laws. So A has to be wrong.
Statistics are mentioned in a parenthetical in paragraph 4, but that’s just there to help make one part of the claim from paragraph 4 more specific. Statistics are ultimately an unimportant part of the claim that the question stem references, so I can eliminate B.
This leads into the claim from the previous paragraph about invariable natural laws, which makes C a really attractive answer choice. However, the question stem is asking me what the claim in the question stem is “most in accord with.” I don’t see how a general claim about natural laws would be in accord with a claim about a specific time period, so C has to be incorrect.
This is correct. It exactly matches my prediction. It also is in accord with the logic of the last paragraph, which states that the science tells us that everything should be completely determined by natural laws: that free will shouldn’t exist.
This goes back to the first question of the set, since the idea of people talking as if inner mental states existed is a reference to behaviorism in paragraph 3. Behaviorism is given in that paragraph alongside quantum mechanics as examples of scientific theories that failed to change the worldviews that they challenged. So the author probably mentions people talking as if they have inner mental states to give an example of something people believe in spite of science that demonstrates the contrary.
The author never blames scientific theories for their inability to change worldviews, so A can’t be right.
Since the author implies that the belief in the question stem is contrary to the scientifically determined theories of behaviorism, it’s unlikely that the belief in the question stem qualifies as a natural law. There’s therefore no way that that belief could be an example of the application of natural law, which makes B incorrect.
C fits my prediction, since the belief in the question stem is one that persists in spite of conflicting with behavioral psychology. C therefore has to be right.
The passage implies that behaviorism may be a new research tradition in paragraph 4, but the belief in the question stem is not a part of that paragraph, so D has to be incorrect.
Since the passage is clearly focused on mythology, this is a main idea question. The author begins by defining myths, then makes the claim in paragraph 2 that “The story of the Western world is, in effect, a story of demythologization.” This is because myths are described in paragraph 1 as “childish things.” So the author’s major contention about mythology has to be something about how it is something childish that needs to be outgrown for us to progress as a society.
The author is fairly anti-mythology throughout the passage, and so is unlikely to feel any nostalgia for myths. I can safely eliminate A.
Religion is mentioned in the passage as replacing myth, but the passage author makes this claim in order to attack religion too in paragraph 5. So the author can’t be focused on B, but must instead be making a broader point about the need to surpass mythology and religion both.
C matches my prediction; the author wants myths to be left behind, and celebrates in the last paragraph that we have “emancipated ourselves from myth and magic.” C is therefore right.
The author definitely defines myths as primitive thought processes, but processes that have to be left behind. The author is more focused on that leaving than on the recurrence of myths, so D is worse than C.
I know that Freud is mentioned in paragraph 5, where the author attacks religion because of its association with myth. More specifically, the author writes for “Sigmund Freud, religion was the obsessional neurosis of children, designed to be outgrown as humanity evolved. Freud believed that in the process of growth, humanity was bound to turn away from religion, and that the beginning of the twentieth century marked the transition from childhood to maturity.” So I’m looking for an answer choice that suggests that religion will eventually die out.
Though the passage first mentions religion as a mental illness, that is a metaphor; Freud is not talking about actual children, and so is not talking about an actual mental illness. A therefore goes further than the passage does. All that the passage states is that Freud believed humanity would outgrow religion, not that humanity would grow to see religion as a mental illness.
B describes religions growing, while the passage describes religions as diminishing. So B can’t be right.
C is meant to be tempting because test-takers might confuse C as saying that the increase in nonreligious organizations will be from members of religious organizations. But C doesn’t say that; it’s entirely possible that membership in these nonreligious organizations could increase and membership in religions increase. So since C does not directly claim that religious membership will decrease, C has to be incorrect.
D matches my prediction, so I can choose it and move on.
Christianity and Euhemerus are mentioned in paragraph 3, where the passage describes the process of demythologization. Euhemerus “proposed that the gods were actual, historical individuals, most often monarchs, whose lives and deeds became wildly exaggerated by the popular mythological imagination.” Since Christians are interested in “denouncing pagans,” the question becomes how Euhemerus helps Christians to denounce pagans. The only way I can square this is that Euhemerus essentially attacks the divinity of pagan gods by suggesting that they are actually just really famous humans who got mythologized at some point. So Christians could use his arguments in order to attack pagan religions. That feels like the right answer.
If anything, Euhemerus would suggest that Greco-Roman mythology had a basis in fact. That basis in fact constituted an attack on the divinity of Greco-Roman gods, since Euhemerus argued that those gods were actually just factual human beings. So A can’t be right.
This idea is associated with Xenophanes, not Euhemerus in paragraph 4, so B cannot be correct.
This is an application of Euhemerus, but one that the Christians described in the passage do not argue. Since Jesus is not mentioned in the passage, C can’t be correct.
D matches my prediction, so it is correct.
It’s not hard to see how much the author dislikes mythology; even though at the beginning the author calls myths “not malicious,” the author also seems to think that myths hold us back. The most telling line that suggests the author’s attitude toward myths is when the author calls them “childish things.” That tells me that I’m looking for an answer choice in which the author acts in a superior manner to myths.
A works as the right answer because patronizing captures the author’s sense of superiority over myths. To be patronizing is also to imply that there is something childish about the subject of the author’s attitude, which is precisely how the author describes mythology.
The author definitely thinks of myths as something in the past, but does not have the positive feelings towards myths that we typically think of as nostalgic. So B has to be wrong.
There is no clear sense that the author is uncomfortable with myths. If anything, the author is comfortable enough with them to judge them as something that civilizations need to exceed. So C has to be wrong.
D is the next most attractive option, because the author comes off as against myths insofar as the author wants to exceed them. But antagonistic is a bit more pointed than the author is; the author clearly doesn’t hate myths, for example, since the author calls them “not malicious” in the first paragraph. That’s enough of a difference from the passage argument for D to be incorrect.
Marx and Freud are both mentioned in paragraph 5, where the author attacks religion for its relationship to mythology. Given that the passage argues that religion holds humanity back because of its relationship to mythology, the author would likely think that Marx and Freud were right, and helped to advance society.
The passage never claims that Marx or Freud were discredited, so A has to be wrong.
B matches my prediction. If Marx and Freud helped people to leave religion behind, and leaving religion behind helped people to advance society, then it is likely that the author would think that Marx and Freud helped encourage Western thought.
The passage only states that Freud and Marx opposed religion. The passage does not suggest that they took concrete steps like starting anti-religion movements, so C has to be wrong.
Since the author seems to think mythology and therefore religion hold Western civilization back, it is unlikely that the author would think Marx and Freud held civilization back. D has to be wrong.
This is a nice straightforward main idea question. I know that the passage begins by opposing the animal and the human mind, and then discussing features of both. The animal mind, paragraph 2 claims, serves animal purposes: “for through the first, it was able to distinguish those things that favor the survival of it and its race, and through the second, it was able to go about in a more efficient manner to secure them.” The human mind, though, is able to do something which is “humanly valuable but biologically useless” (paragraph 3). The author calls this an effort to make the human mind “detached,” which the end of paragraph 3 elaborates on: “We admire anyone whose reason is capable of more than scheming, whose logic is not the mere rationalization of desires.” In other words, humans aspire to thinking that is not about mere survival: the selfish desire to keep living. Paragraph 4 onwards claims that this tendency to avoid selfishness “is a vital liability” because “It puts its possessor at a disadvantage in dealing with those whose intelligence faithfully serves their purpose by enabling them to scheme for their ends and to justify to themselves their desires.” In other words, this unselfish and perhaps “objective” mindset puts one at a disadvantage when one is up against people with an animal mindset: people who want to live at all costs, and are willing to do anything to do so. That idea carries through the passage’s argument about civilization, so I’m looking for an answer that mentions the downsides to unselfishness or objectivity.
This is a pretty brutal passage. A is meant to be attractive because the passage calls the human mind’s weakness “a quest for that self-justifying beauty,” but the passage is more focused on what is being called beautiful than concepts like truth and beauty themselves. A has to be wrong.
Because A distorts the passage logic, I’m on guard for how B does too. At no point does the passage say that intelligence is biologically useless; the author is making a finer distinction than what B proposes in arguing that human intelligence can lead to thinking that is biologically useless. B is therefore wrong.
C makes sense as the right answer. “Unbiased” is a synonym of “detached,” and the passage argues that the attempt to be unbiased towards one’s biological needs and wants puts individuals and nations at risk. I can choose C and move on.
D, yet again, distorts the passage logic. The passage may be suggesting that we need to learn something from animals, but it at no point suggests that we are most human when we behave like animals. The author maintains a pretty strong animal-human distinction throughout the passage: one that is too strong for D to be correct.
The last question helped me clarify what the author’s central argument is: that unbiased thinking can negatively affect human chances at survival. The other way to read that main idea is that animal thinking can be better than human thinking when it comes to survival. So I’m probably looking for an answer that says that there are survival benefits to unbiased thinking, but weakeners can take many forms, so I am most strictly looking for answer choices that would challenge central passage claims.
A can’t be right. The author’s problem is that human tendencies towards objective or unbiased thinking leave people prey to those who focus more on survival, and are willing to do anything to survive. Civilization is equated with this more objective human mind in paragraphs 5 and 6, and so it would both be in line with the passage that highly developed civilizations would be peaceable, and that the author would also think that this is a liability: “One after another, the great nations of history have founded on aggression the civilization that then supported for a time, but for a time only, great periods of human culture, that flourished at their height just as the substructure crumbled. Animals made humans possible, and conquerors prepared the way for poets and philosophers, but neither poet nor philosopher can survive long after the conquest.” Since A is in line with the passage argument, it would likely strengthen the argument, not weaken it.
B sounds like it contradicts the passage argument about how “we admire anyone whose reason is capable of more than scheming, whose logic is not the mere rationalization of desires” (paragraph 3). But the passage itself argues that we should not admire this mindset. B therefore supports the author’s main argument, since it suggests that there is some benefit to being selfish. If, after all, one of the benefits to being unselfish is that you are admired, then saying that selfish people are also admired subtracts that reason to be unselfish. B is therefore incorrect.
C isn’t what I was expecting, but it does affect a key part of the passage argument: that the animal mindset is one of selfishness, first and foremost. The passage argument actually relies on the idea that we should emulate animals quite a bit, so if animals behaved unselfishly—which is to say, if they behave like humans—then that weakens the author’s argument. Note that C does not disprove the author’s argument; one could argue, for example, that the passage isn’t really about human and animal, but unselfish vs. selfish thinking. But the question is not asking for a choice that destroys the passage argument; the right answer need only make the conclusion less likely, or less convincing.
I don’t see why D would weaken the argument. I know D already because paragraph 3 states that unselfish people are admired because their “logic is not the mere rationalization of desires.” I also know that the author thinks that unselfish forms of logic can be bad. So at best, D is in line with the passage argument, and so should be eliminated.
This question is asking me to use passage logic to explain the situation in the question stem. Because that situation involves an opposition—and one between two people of different levels of intelligence—that brings to mind the opposition between animal and human minds. The passage suggests that animals may be less intelligent than humans, but are ultimately more successful at survival. So perhaps the person of average intelligence is more willing to be competitive—to do whatever it takes to survive—than the more intelligent person.
While the passage mentions art and philosophy, the situation in the question stem does not clearly involve either, so I can eliminate A.
The passage is very clear that intelligence and competitive success are related. Namely, the passage argues that humans, with their higher intelligence, put themselves at a competitive disadvantage. So B has to be incorrect.
This is getting closer, insofar as it is suggesting that there may be a problem with the more intelligent person, but the passage never says that unselfish humans despise material success. The author’s focus is more on the ways that unselfishness can distract one from focusing on survival enough to endanger one’s survival.
D is the last answer left, and it luckily matches my prediction. I know that the passage suggests that “higher” intelligence can keep one from focusing on survival, so it makes sense that a smarter person might not be as competitive as the person of more average intelligence.
This question is asking me to determine what answer choices can be argued against with a counterexample. Another way to think about this, though, is that the wrong answers will contain some element that is too abstract or ambiguous to directly argue against: that needs some clarification to pin it down. The right answers will be quantifiable enough that some demonstrable fact could disprove it.I. The intelligence of poets tends to paralyze natural impulse. It’s hard to see what clear example could argue against this. I’d need to come up with a poet whose intelligence does not paralyze natural impulse, but what exactly does that mean? How would I measure that? I’d need a lot of clarification to make an example to counter I, so I is likely incorrect.
II. Transforming means into ends is the most characteristically human effort. II is also difficult to argue against because it’s not making a quantifiable claim, despite using the word “most.” I would need to either find an example of a “human effort” that is more characteristically human, or an animal for whom transforming means into ends is more characteristic than it is for humans. But again, how would I prove that some other trait is more human than any other? These questions tell me that II has to also be wrong.
III. The great nations of history were founded on aggression. III is the last option left, so it has to be correct, and it is. It is at least conceivably possible to come up with counterexamples to III; I would just need to imagine nations that were not founded on war. I cannot do that with either I or II, so III’s rightness also helps me confirm that I and II are wrong.
II is incorrect, so A is wrong.
This is the only correct option, so I can choose it and move on.
Both these options are incorrect.
I is incorrect.
The author’s argument depends on there being an opposition between selfish and unselfish thinking. But if unselfish thinking was secretly selfish—as the question stem suggests—then perhaps those terms are not really opposed at all. That should be enough for me to find a right answer.
A works for this question. The “animal” and “human” versions of intelligence are treated as “selfish” and “unselfish” thinking, so to attack the distinction between “selfish” and “unselfish” thinking is to also attack the distinction between “animal” and “human.” I can choose A confidently.
I can eliminate B right away because the research in the question stem is talking about the reality of thinking, while B talks about the perception of thinking. It’s possible for people to still value unselfish thinking even if it is impossible to ever think unselfishly.
I don’t see how research showing that we can’t ever really be fully intellectually detached would suggest that intellectual detachment is biologically useless. I suppose the idea is that something which is impossible should be useless, but the arguments about why intellectual detachment is biologically useless have to do with specific aspects of intellectual detachment, and not with whether or not that mindset is possible. I can therefore eliminate C.
If anything, the research would weaken the claim mentioned in D, not strengthen it. If the author thinks that unselfish thinking is a biological vice, and the research claims that unselfish thinking is actually selfish thinking, then the one use of intelligence that the passage claims is a biological vice suddenly is not one. So perhaps there are not uses of intelligence that are biological vices. That makes D incorrect.
I know that newscasters are mentioned at the end of paragraph 1, where the author notes that newscasters say “’stay tuned’ is the eleventh and commanding commandment.” So this question is really asking me why newscasters say “stay tuned,” a notorious newscaster phrase. This quote occurs at the end of the paragraph, and so it is explained by the sentence before: The reward of being a viewer depends on staying passive – if we are moved to leave the viewing chair we may miss the next program.” The right answer, then, has to say something about how newscasters want viewers to stay passive: to keep on watching.
A is almost a quote from that first paragraph, so I can choose it and move on.
The discussion of television news actually says the opposite: that the news is frustrating, because so much of it is given to us, and because “it seems always about to tell us something.” Frustration is the opposite of catharsis, so B has to be wrong.
Like B, C says the opposite of what the passage does. The whole point is to keep viewers watching. So C is also wrong.
The first paragraph mentions D—that “the segmentation and ‘personalization’ of the ‘news’ actually” confuses “us with their discontinuous gush”—which makes D a tempting enough answer choice that I need to be sure it is wrong. In this case, D has to be wrong because the first paragraph only claims that D is what the news does; it is not what newscasters try to do. That’s enough of a difference from the passage to make D worse than A.
I know that the passage talks about reality constantly, but it first does so right at the beginning of the passage: “Television is not the ‘dream factory’ which Hollywood was once said to be by dour sociologists: it is a reality factory. It is truer to life than life is.” The author explains what this means later in the passage. This happens first with the example of Kojak, a television policeman who “at the peak of his powers and popularity, gave police officers – as do and did other realistic programs – an indication of how they should behave (and how the public would expect them to behave) if they were to maintain credibility” (paragraph 3). It then gets explained more starkly in the last paragraph: “Television writes our scripts and it gives us back our language in a verisimilitudinous revision, docked of amateurish or embarrassing passions or obsessions which might cause our audience to switch off.” In other words, what television does is provide a model for people’s speaking and behavior. So the real-life argument described in the question stem would likely make for bad television because it would be “amateurish” or too passionate or obsessive: would deviate too much from what is expected.
A is tempting because the real-life arguers would be less convincing, but they wouldn’t be less convincing because they lacked professional training. The passage is less interested in what actors do—actors are barely mentioned—and more in what television as a concept and a technology does. So A has to be wrong.
Precision doesn’t seem like the problem these arguers would face either. Precision is actually mentioned in the paragraph as one of television’s weaknesses: “A lack of articulacy is a badge of sincerity; grammar smacks of premeditation” (paragraph 1). Since television is not articulate, it is unlikely that the real-life arguers in the question stem would make for bad television because they too are inarticulate.
Passivity is mentioned as a quality of viewers in paragraph 1, but it’s not as if activity is discussed as a quality of actors in the passage. I can quickly eliminate C.
D is the last answer, and it has much of what A lacks. It directly refers to the real-life arguers as “passionate”—something that television edits out—and it explains that the real-life arguers would be unrealistic because of something to do with how television itself operates. That’s enough to make D correct.
Characters who recur are mentioned in paragraph 1: “What has been said before – ‘characters’ we have seen before, advertisements we ‘love’ – may well be the evidence that originality (what has never been said before) has scant future on the box.” In other words, the fact that we are so familiar with tropes and characters on television may suggest that we do not really care much for originality.
A matches my prediction, so I can choose it and move on.
I can see why someone might think that repetition would have a numbing effect, but this point is made after the quote referenced in the question stem, and is a slightly different point, so B can’t be the right answer.
This is the author’s overall point, but there’s no clear way to relate the repetition of characters to this claim. The author is instead making a more localized point with the quote referenced by the question stem in the process of arguing C. C therefore has to be worse than A because the overfamiliarity described is not used as direct evidence for C.
This idea precedes the discussion of repeated characters; if anything, then, D is either evidence for the claim in the question stem, or is a tangential argument to that claim. I can therefore eliminate D.
Here’s a straightforward weakener, so I have to evaluate each answer choice and see what new piece of evidence would challenge something from the passage.
Given that this passage is about the power of television, it would be perfectly in keeping with the passage for people to like television, critics included. A is therefore incorrect.
B resembles the passage description of television news in paragraph 1, which the author claims is guilty of “hosing us with ‘information.’” “Hosing” here means overwhelming us with information: something that “fast-paced images and simple language” would do. So B too has to be incorrect.
C catches my attention because paragraph 1 says that television is not articulate—“A lack of articulacy is the badge of sincerity”—and because the last paragraph says that television removes “embarrassing passions or obsessions.” That’s enough of a challenge to parts of the passage to make C correct.
I know from my work on previous questions that television does reuse a lot of characters and plotlines, so D is in line with the passage argument, which means it cannot weaken the passage argument.
This feels like it is either a reference to the main idea—the idea that television makes what it presents feel real—or a reference to what television does to viewers: keep them passive and watching (paragraph 1). I’ll be on the lookout for either as the right answer.
This is definitely not correct. The last sentence tells me that television should never hit us with the “full effect’ of emotions; it instead curates our language and mannerisms to make them feel more natural. So A can’t be right.
B matches one of my predictions—that a good program will keep viewers passive and watching—but since the passage does not really explicitly say what a “good program” is, I need to be surer of this answer by eliminating the other answer choices.
C is really close to A: close enough that I can safely eliminate it.
And D contradicts the claim in paragraph 1 that television tends to repeat itself. Specifically, D contradicts the idea that “Television convinces us by immediacy and by repetition, not by structured argument or oratorical exposition.” And since there is no specific reference in the passage to novelty making for good television, I can safely eliminate D and choose B.
This basic question is made really difficult by a challenging passage. There doesn’t seem to be an obvious line of inquiry in the passage, so one way to discern the central claim of this passage is to think through what the passage paragraphs have in common. The passage begins with the writer marveling at what happens around a sunflower. That transition into the second half of the passage, which shifts to talking about “other summons”: how some things in nature seem coordinated in ways that feel mysterious. That doesn’t quite feel like enough for me to judge what the right answer will be, but that will hopefully be enough for me to determine what the wrong answers will look like.
There’s nothing in the passage about primitive peoples’ beliefs, so I can safely eliminate A.
There is definitely some talk about ecosystems and its components, but the perspective in this answer choice is wrong. The passage describes a poet or a natural philosopher speaking at best; there is not enough scientific language or perspective in the passage for B to be correct.
C makes some sense, because the perspective is definitely poetic. I can also pick out parts of the passage that C resembles, such as the author marveling over how the sunflower was “turning its face daily, always toward the sun.” Still, I’m very unsure of C, so I should try to eliminate D to confirm that this is correct.
Luckily, D is definitely wrong because it makes the same mistake B does. There isn’t enough of a scientific perspective or scientific language for the passage to be written by a biologist.
The most prominent biological cycle that is described in the passage is in paragraph 5: the discussion of the bamboo plants, and how all of them bloom on the same day, no matter where they are in the world. The only explanation the passage offers is: “Some current we cannot explain passes through this primitive life. Each with a share of communal knowledge, all are somehow one plant.” So the right answer will perhaps mention something about how these plants share knowledge.
The passage says that these plants are “communal,” not competitive, so A has to be incorrect.
B sounds communal, but I am a bit worried about it because paragraph 5 only talks about “communal knowledge”; that could mean knowledge that is communicated, but it could also mean knowledge that the plants all have. So I’ll double check the other answer choices to see if I can eliminate them.
The example of the bamboo plant does not clearly involve adaptation. It describes plants doing the same thing in different places, which is not really similar enough to plants adapting to different environments to make C correct.
D is actually a little too similar to C to be correct. The example of the bamboo plants states that all the plants flower on the same day no matter what environment they are in; that actually implies some insensitivity to the environment. That means while B is not a great answer, every other answer is clearly wrong, so B must be correct.
The horse and sandstorm are mentioned in paragraph 3. That paragraph begins with a claim: “Changes also occurred in the greater world of the plant.” The horse and sandstorm are mentioned quite factually: “One day, rounding a bend in the road, I encountered the disturbing sight of a dead horse, black against a hillside, eyes rolled back. Another day I was nearly lifted by a sandstorm so fierce and hot that I had to wait for it to pass before I could return home. It swept away the faded dried petals of the sunflower. Then the birds arrived to carry the seeds to the future.” Again, this is a challenging question to imagine the answer to, so I’ll have to hope that I get some help from the answer choices, and that keeping the quote in mind will help me recognize the correct answer.
While there is a general sense of mystery in the passage, the horse and sandstorm paragraph doesn’t clearly refer to mystery beyond human comprehension, so I can eliminate A.
B is meant to be tempting because the sight of a dead horse is likely to be thought of as ugly by most, but the paragraph doesn’t mention beauty or ugliness, so B has to be wrong.
C is meant to be attractive for the same reason B is: because it plays on our instinctual feeling that a dead horse is a bad thing that can only come about because nature is cruel. But again, like C, the paragraph doesn’t say anything about how cruel nature is, so I can eliminate C.
D has to be correct via the process of elimination. I wouldn’t have chosen it on its own, but I can see why it is correct. The paragraph is meant to describe a larger version of the perpetual drama described in the previous paragraph, and the sandstorm does play a role in the drama around the fire: “It swept away the faded dried petals of the sunflower. Then the birds arrived to carry the seeds to the future.”
This is a passage check question; all I can do is check and see which answer choice occurs in the passage.
This is definitely assumed in the example of the bamboo plant. My work on a previous question tells me that these plants communicate with each other, but in a way that the author—and humans—can’t detect. So A is assumed by the passage.
This sounds like the last paragraph, where the author mentions the “people who came before me and their knowledge of the placement of stars, people who watched the sun long and carefully enough to witness the angle of light that touched a stone just once a year.” So even though B is given with extreme wording, the openness here actually makes it a good assumption: something unstated in the passage. That means B has to be incorrect.
I suppose C has to be incorrect because the author seems to want to be in harmony with other species. That’s perhaps clearest in the 6th paragraph, where the author senses “something like a heartbeat, a hardly perceptible current that stirred a kinship and longing in me, a dream barely remembered.” Kinship here could definitely mean being in harmony with, so I can safely eliminate C.
D has to be right, and it clearly is because it goes against the logic of the passage. The passage focuses on how much the author is in awe of nature: how much mystery there is in natural processes. It would be counterproductive to the author’s work to suggest that at some point, we would know enough about nature to stop being curious about it. D is therefore correct.
Another broad question, but this time I’m helped by the answer choices, which tell me that I need to describe the ideal relationship between humans and animals in abstract terms. I know that the author seems to be in awe of nature, and therefore of animals. Most importantly, the author seems eager to claim ignorance, and a desire to learn from them. That should be enough for me to find an answer.
So this choice says that humans are benefactors, and animals are recipients: that humans give something to animals. The passage doesn’t clearly suggest that the author, or humans in general, give anything to animals, so A can’t be right.
B is immediately wrong because, while the author studies animals, that study does not seem to resemble scientific study. There’s a lot of emotion and subjectivity in the passage: more than scientists are supposed to have when studying a subject.
C sounds like a good answer choice because it aligns with my prediction. The author is eager to learn from nature, which makes the author (and humans) students with animals (and nature) as teachers.
D has to incorrect because it is too similar to A, since parents are (hopefully!) very much benefactors for their kids. D therefore has to be wrong: both because A is wrong, and because if D was right, A would likely also have to be right, and I can’t have 2 right answers. C has to be correct.
The question stem’s mention of the fifteenth and sixteenth centuries is meant to make me waste time on this question. Even though the passage begins by mentioning the “second half of the sixteenth century” in the beginning of the passage, all of the art mentioned in the passage is made during those two centuries. So this question is really just asking me to discern what subjects all the art in the passage has in common. Paragraph 2 describes ancient mythological figures like Neptune, while paragraph 3 and 4 bring up biblical figures like David, of David and Goliath fame. That should be enough for me to choose a correct answer.
Ammannati’s exploits are only described in relation to the art in paragraph 2; they do not have anything in common with the art described in paragraphs 3 and 4. That means I can’t justifiably say that the Florentine art of two centuries often used A as a subject.
B, though, matches my prediction, since it captures both the artistic subjects mentioned in paragraph 2, and those mentioned in 3 and 4. I can therefore choose B confidently.
The Counter-Reformation is only mentioned towards the end of paragraph 2, as part of an explanation for why Ammannati changed his mind about his statues. I don’t even know if the Counter-Reformation made for artistic subjects based on the passage, so I can eliminate C.
Rubens is mentioned in paragraph 2, but no works of Rubens are discussed in the passage, so I can safely eliminate D. B has to be right.
The David is discussed in the last paragraph, which I remember mentions something about a “turning point in Michelangelo’s art: from the ‘sweet’ to the ‘terrible’.” So I know that the right answer will have to be some synonym of “sweet,” and the opposite of “terrible.”
Donatello and Michelangelo are contrasted at the beginning of the paragraph. I know the passage describes Donatello’s statue as “tragic,” which does not clearly resemble “sweet,” so I can eliminate A.
I also don’t know what “classical” has to do with “sweet,” so I can eliminate B.
“Calm” isn’t obviously “sweet,” but it is the opposite of “terrible,” so that makes C tempting. I would just check D to make sure it’s clearly wrong and then choose C.
This is incorrect because David is a biblical figure, not a mythological one. The passage makes a distinction between them in the second paragraph; mythological figures like Neptune were seen as unacceptable by Ammannati, when Ammannati became more Christian.
Damage to statues in the square is barely mentioned in the passage, so my best strategy is to go through each option and just check and see if it is mentioned in the passage.I. Replacing the original statue with a copy I is mentioned in the last paragraph with regards to the David: “In 1873 the original was placed in the Academy and this statue is a copy.” So I is correct.
II. Moving the statue into one of Florence’s museums Paragraph 3 mentions that Donatello’s statue was moved “into Palazzo Vecchio.” I don’t know if that’s a museum, but it would make sense to move a statue to a museum to protect it, so that’s enough for me to choose II.
III. Erecting a protective barrier around the status I don’t see any examples of barriers being placed around statues, so III is incorrect.
I is correct, but so is II, so A has to be wrong.
II is wrong, so B is wrong.
C is the only answer that contains both answers I’m looking for, so I can choose C and move on.
And D is wrong because III is wrong.
I can only answer this question about a very specific part of the passage by having the text of this passage in mind: “Some of the sea gods and goddesses, with their stylized elongated bodies, are mannerist in style; but the satyrs, vital and full of movement, are naturalistic in a warm and sensual way.” So being “mannerist in style” must involve “elongated bodies,” and not involve “warm and sensual” depictions of bodies.
A is how the satyrs are described in the quote. But the satyrs are introduced with a “but” after the author mentions mannerism, which means that the satyrs are contrasted with mannerism, or are at least differentiated from mannerism. A therefore cannot be correct.
This is a reference to paragraph 4, and the description of changes to Michelangelo’s style. Since this happens way after paragraph 2, I can safely eliminate B.
This is Ammannati’s later description of his sculptures, but the passage does not clearly suggest that he is referring to just the mannerist figures. It would make sense that he would be referring to the satyrs—which are not mannerist—since they are “sensual,” which is a synonym of “libidinous.” Since I can’t directly link mannerism to C, C has to be incorrect.
D definitely matches the description of the sea gods and goddesses, which are mannerist in style according to paragraph 2, so D has to be correct.
The question is asking me what answer choice would be weakened by passage information. To answer this, I need to be sure that I know what the reference to “Il Biancone” means in the passage. The quote comes from paragraph 2: “The fountain probably was meant to celebrate Cosimo’s maritime successes (in fact, Neptune does bear a resemblance to the duke): but it is not the colossal and rather clumsy figure of the god, whom the Florentines rather wickedly call ‘Il Biancone’, ‘the big white one’, which attracts one’s interest.” Since the reference is called “wicked,” the Florentines likely mean the term “Il Biancone” to be an insult. That should be enough for me to find a right answer.
I can’t see why the reference would suggest that Florentines were not well versed in mythology. “Il Biancone” is a reference to the appearance of the statue, and is not clearly indicative of Florentine knowledge of mythology. I’d eliminate A.
B feels good as the right answer, since I predicted that “Il Biancone” is an insult. Calling the statue an insulting name would definitely challenge the idea that people admired the statue.
“Clumsy” is a negative word. Since “Il Biancone” is also implied to be a negative word, it’s more likely that people calling the statue “Il Biancone” supports C. That makes C incorrect.
“Florentines” doesn’t clearly include Cosimo I, so I have no reason to see D as correct. I can therefore eliminate D and choose B confidently.
This question asks me to identify what evidence the author provides for a point. Such evidence will usually be found immediately before or after a claim is made. Popular songwriters are mentioned in paragraph 1 in such a way that it makes clear that the evidence will follow the point: “Driven off by too much that is merely commercial – often shoddy imitation of authentic originality in the realm of the popular – we fail to notice that popular song writers . . . can be dedicated, energetic poets more interesting than many of the weary sophisticates, true-confessors, and randy academics we encounter in the “little magazines.” I can stop there because the “and that drugstore fiction” which follows that quote signals that a related but different point is being made. So I know that the point about popular songwriters is argued with a comparison to writers that are perhaps well-regarded and academic, but are less interesting than these songwriters.
Literature teachers aren’t quoted here, so there is not enough testimony when talking about popular songwriters to make A correct.
I don’t see any examples of popular song lyrics, so I can eliminate B.
I know that the passage mentions “academics” when arguing the point about popular songwriters, so that makes C compelling. And the fact that the quote calls popular song writers “dedicated, energetic poets more interesting” than the list at the end of the quote suggests that the author is comparing poets to poets: that “sophisticates, true-confessors, and randy academics” are all poets. That’s enough to make C correct.
Coleridge is mentioned later in the paragraph, but no analysis of Coleridge is provided, so D has to be incorrect.
This speaks to the main idea of the passage. The author spends the first paragraph pointing out the phenomenon the author is curious about: why it is that literature teachers do not teach popular entertainment. The second paragraph offers an explanation: that there might be some pressure to teach students something objective, instead of providing them access to literature “because they give joy, the incomparably rich experience we ask and expect of all true art.” The third paragraph then clarifies the author’s perspective: that it is a problem that “At all levels, . . . novels, short stories, and poems have for years been taught not as experiences that can delight and enliven the soul but as things that are good for us, like vitamin C.” So the right answer has to mention that teaching fiction and poetry should provide students with experiences.
This is what the author claims currently happens in paragraph 2. It is thus not what the author says should happen because it is what the author worries about. So A has to be wrong.
B is the same answer choice as A: teaching major literary themes, the author argues, is akin to trying to teach students “objectively.” That makes B incorrect.
Textual analysis also falls in the same category, since “analysis” is very similar in concept to an “objective” approach to study. So C also has to be wrong.
D matches my prediction that the right answer has to reference experience, so I can choose D and move on.
“Drugstore fiction” is mentioned in the first paragraph, where I know the author sets up an opposition between the texts that teachers choose for literature classes and “simple entertainment”: that is, the stuff that people enjoy on an everyday basis. It is an opposition between texts that people enjoy, and those that people do not. So I would imagine that, since “drugstore fiction” would fall into the camp of “simple entertainment,” that teaching such books would lead to happy students.
I can eliminate A because the passage doesn’t focus on reading skill levels. The passage is more about the texts taught in classes, and the reasoning that underlies those choices.
I don’t remember anything about committees in the passage, so that already makes B suspect. The author is also making a recommendation, but there is no argument in the passage which suggests that literature courses would make the change that the author recommends, so B has to be wrong.
C matches my prediction, so I can choose it and move on.
This is a reference to paragraph 2, and how literature classes are currently taught. Since drugstore fiction is not currently taught in literature classes, there is no obvious basis in the passage for me to believe way that teaching drugstore fiction would help students understand literary terms. D therefore has to be wrong.
To put it simply, the author thinks that literature is rewarding when it is focused on experiences, and less rewarding when it is treated “as things that are good for us, like vitamin C.” The author finds current methods of teaching literature in an objective way as less rewarding, so the right answer will have to refer to that.
A matches my prediction, since “critical analysis” is a fairly objective way to approach literature: one on par with the approach the author dislikes in paragraph 2. I can choose A confidently.
“Useful” is another reference to paragraph 2, and the approach the author dislikes because it is less rewarding: “It is often tempting to treat life-enhancement courses as courses in useful information, putting them on the same ‘objective’ level as courses in civics, geometry, or elementary physics.” So B can’t be correct.
If anything, the author is curious about the potential of this literature. The first paragraph makes it clear that much of this kind of entertainment is bad and so would not be rewarding, but the passage ultimately argues that avoiding such entertainment also seems to mean missing out on literature that gives us enriching experiences. So the author actually says the opposite of C: that it might be more rewarding to teach some commercial entertainment.
There’s that word “experience” again. I know that the author is for something like D, and so the author would not find literature that offers a life-enhancing experience to be less rewarding.
“Science fiction” is listed in the first paragraph as one of the kinds of popular entertainment that literature teachers do not teach. So if students got to read Asimov in class, that would suggest that literature teachers do teach popular entertainment.
I don’t have a good basis to choose A, simply because I have no idea if Asimov’s work is simpleminded or not. So A has to be incorrect.
I also can’t choose B because I don’t know what prescribed curriculum objectives are being referenced here.
The passage never says anything like C, so C is immediately a bad answer choice in my mind. Also, I don’t know if choosing Asimov demonstrates someone’s understanding of literature. So I can eliminate C.
D matches my prediction, since teaching Asimov, a science fiction writer, means that literature teachers do teach popular genres. And while D is not disproven by the situation in the question stem—it’s possible to be prejudiced against something that you end up teaching—teaching Asimov does challenge D: does make it less likely, and argue against it. D is therefore correct.
This is really a main idea question, since the question is asking about the primary significance of the primary subject of the passage. The passage begins by saying that the “place” of Neandertals is “mysterious,” and then goes on to explain why: because while they physically differed from humans, and did not advance past a certain level of technology despite existing for a long time, they also resemble humans in some ways, such as their willingness to bury their dead. Since the author does not seem to come up with a clear answer to the question asked in paragraph 2—do we “Call them Homo sapiens neandertalensis and acknowledge them as direct ancestors of modern humans? Or type them as Homo neandertalensis and more distant relatives, members of a separate species outside our direct ancestry?”—I can safely predict that the right answer has to mention how difficult it is to classify Neandertals.
A matches my prediction, so I can confidently choose it and move on.
This sounds almost as attractive as A does, but paragraph 2 actually calls this into question. Since the passage questions whether Neandertals actually were premodern humans or not—“members of a separate species outside our direct ancestry”—B can’t be correct.
The passage only mentions the use of stone tools in one part of the passage, and only refers to Neandertal stone tools in order to make a point about whether or not Neandertals were our ancestors, so C is too limited to be an explanation for the “primary significance” of Neandertals.
D is rhetorically the same answer as C, insofar as both refer to arguments that suggest that Neandertals may have been prehumans. But since D is part of a larger argument about Neandertals, it too cannot be the primary significance of Neandertals. D is therefore incorrect.
This question quotes the start of the last paragraph, where the author uses stone tools to note how Neandertals and humans differ. In particular, that paragraph points to the slow pace of stone tool development by the Neandertals: “For the million years after the appearance of tools in the record, they remained crude in structure and limited in variety […] Only about 200,000 years ago did the pace begin to change […] When the Neandertals came onto the scene, they further refined this technique. Nevertheless, no further innovations were introduced for more than 50,000 years, when the modern human era began.” The passage here seems to be drawing a parallel between the stone tools and the rate of development of whoever made them. That seems like a good explanation for what the quote in the question stem means.
Nothing in the last paragraph mentions needs for better tools, so A can be quickly eliminated.
The passage also does not talk about patterns of development; it only mentions when development sped up and slowed down. B therefore can’t be right.
I don’t see anything about radiocarbon dating in the last paragraph, where stone tools are mentioned, so C has to be wrong.
D is the last answer, and so it has to be right. It also matches my prediction: that the rate of stone tool innovation says something about the innovator. Just as the fact that humans were able to rapidly innovate on stone tools says something about humans, the fact that Neandertals eventually stopped innovating on stone tools says something about them.
This is an odd question, if only because the mention of Ethiopia and the timescale mentioned doesn’t seem to relate to the passage much. Homo erectus is mentioned in paragraph 1, where the author just says that Neandertals lived between the times of Homo erectus and Homo sapiens sapiens. Somehow, this information should help support one of the answer choices.
I can’t tell whether or not I should eliminate or choose A. On the one hand, having Homo erectus pre-exist Neandertals so at least makes it possible that Homo sapiens sapiens and Neandertals have a common ancestor in that Homo erectus. On the other hand, it’s also possible that Neandertals are unrelated to either. The question is only asking me for something which “increases the likelihood” of one of the answers, but I’m just not seeing enough to clearly suggest that A is right or wrong. So I think my best bet is to hold onto A and see if another answer choice is better.
The passage actually never refers to apes as ancestors to humans, so B is not a question that the author has asked in the passage. That makes B incorrect.
I don’t see how the new information in the question stem helps support this. The passage already talks about Homo erectus and the possibility of multiple family trees; knowing that there was some Homo erectus in some part of the world doesn’t seem to make the idea that multiple human family trees exist more or less likely. So I can eliminate C.
D is also very strong: so strong that the information in the question stem would have to talk more directly about Neandertals to make D correct. Since the question stem does not do this, D is incorrect, and A is, improbably, the correct answer.
This is a two-step question. I need to first determine how the author supports the claim in the question stem, and how effective that evidence is. The easier versions of this question type typically don’t require me to do the second step; only one answer choice will match with the passage argument. This claim is given at the end of paragraph 6, which means that support for the claim has to either come in the next paragraph (which is less likely), or before it. The next paragraph starts with a new point—“Stone tools clearly signal the pace of change in human prehistory”—so that doesn’t support the author’s claim about Neandertal artifact manufacturing. The sentence before it—“Reindeer, wooly rhinos, and mammoth provided not only meat but also hide for clothing and bone for building shelters, as wood and other plant resources were scarce or absent”—doesn’t even talk about artifact manufacturing; it’s actually support for the claim made at the start of the paragraph. So I don’t see any actual support for the claim referenced in the question stem, which means that the support is weak: limiting me to A or B. And I need an answer choice which explicitly says that this claim is unsupported.
A is incorrect because the passage doesn’t even mention that the tools were few and crude. This choice is meant to tempt me by encouraging me to assume that the passage discussion of artifacts includes stone tools in the next paragraph: where Neandertal tools are implied to be crude relative to modern human innovations. But that’s a dangerous assumption to make. The actual quote about Neandertal artifacts in paragraph 6—“The resourceful Neandertals also manufactured a wide range of artifacts with which to tackle their daily chores”—arguably includes stone tools, but since stone tools would be a subset of artifacts in that reading, I can’t be sure that what I say about stone tools would apply to all Neandertal artifacts. Finally, the description of a single kind of artifact doesn’t clearly argue that the Neandertals “manufactured a wide range of artifacts,” so even with that generous reading, A has to be incorrect.
B matches my prediction, so I can choose it and move on.
I should eliminate C immediately because I know the evidence for the claim is weak, not strong.
D has the same problem as C, and so should also be eliminated.
This is a strange analogy question, which is asking me to extract a principle from the passage in order to describe the passage arguments in terms of computers. That’s a complex task, so I need to take one step at a time. First, I need to discern on what basis the author argues that Neandertals were human or not. The clearest place to look for that is paragraph 4, where the author states that “Some clues to their potential ‘humanness’ do exist.” The example given is that “For the first time in history, the Neandertal people performed ritual burials.” Nothing else after this point is clearly stated as evidence for Neandertal humanity; later points could be read in that way, but the only clear argument made after that is in favor of Neandertal evolution. So I’m looking for an answer choice which resembles ritual burials: an action which is not strictly necessary, but has some kind of extra significance beyond the act itself.
I suppose this is meant to echo the end of paragraph 3, where the author states that Neandertal brains were larger than humans. We often think of brain size as indicative of closeness to humans, but the passage does not make this claim, and brain size is not listed as indicative of humanness in the passage, so I can eliminate A.
B is meant to echo the last paragraph’s discussion of stone tools. But I can eliminate B for the same reason I eliminated A: because while we often think that tool manufacture is a uniquely human trait, the passage does not make this claim, so B has to be wrong.
I don’t even know what C refers to. It might be referring to paragraph 6, where the author talks about what Neandertals hunted, but that also doesn’t refer to their humanness, so C has to be wrong.
I’m not sure I would have chosen D on its own, so I’m glad that I could eliminate the other three answer choices. “Ritual burials” are described in paragraph 4 as religious: “Some prehistorians speculate that these bones were covered with meat at the time of the boy’s burial and were included as sustenance for his journey to the next world.” That’s more similar to the passage argument about Neandertal humanness than any of the other answer choices, so I can feel good about choosing D.
The passage argument about color-word synesthesia, as crystallized in paragraph 2, is that color-word synesthesia is actually “color-grapheme-association” rather than “color-word association” because it occurs “not between meaning and color, or sound and color, but between the visual appearance of the first letter of the word and color.” So if I’m going to weaken this answer choice, I need an answer that either directly challenges the idea that it is the appearance of the first letter of a word which encourages a synesthetic response, or that there is some other cause for synesthetic responses. The other place where the passage argument talks about color-word synesthesia is paragraph 3: “Grapheme perception is not present at birth and only begins to develop when a child is learning to write. This fact opens up the possibility that color-grapheme synesthesia emerges during a critical period of maximum plasticity in the visual system, when it is involved in learning to link letters with sounds and strings of letters with objects.” So another possibility for a weakener is a choice attacks the idea that synesthesia develops early: during a specific period of brain development.
A sounds like it might weaken the argument, since it suggests that there might be some other explanation for synesthesia, but this other explanation actually doesn’t contradict or challenge the passage explanation for synesthesia. It is entirely possible that what the passage argument claims is true and A could be true at the same time. So since A does not actually undermine the passage argument about color-word synesthesia, A has to be incorrect.
B resembles my second prediction: the one based off of the claim regarding synesthesia developing when people are young. B is a pretty definitive contradiction of that claim, so I can choose it and move on.
C is actually in line with the passage argument in paragraph 3, which claims that color-word synesthesia might develop when one is just beginning to learn how to read.
An answer choice which says that something does not result in synesthesia cannot weaken the argument unless it specifically involves those causes for synesthesia that the passage argument advocates. The passage is primarily focused on claiming that synesthesia occurs at a specific developmental stage and involves specific triggers; it does not focus on how damage to the visual cortex might result in synesthesia. D describes an experiment done to help support the explanation of synesthesia that the passage advocates, but it therefore does not directly touch on the actual causes of synesthesia discussed in the passage. D would therefore not weaken the passage argument.
This refers to paragraph 2 again, which states that “The link is not between meaning and color, or sound and color, but between the visual appearance of the first letter of the word and color.” So the right answer has to just have two words that start with the same letter.
These don’t start with the same letter.
These do, so B is right.
Different starting letters makes C wrong.
D runs into the same problem, so it’s wrong.
This is an odd question type, since it is asking me to come up with an explanation for why the human visual cortex might develop in a way that allows for synesthesia. Specifically, the passage state that specific colors and shapes trigger certain neurons. Since the answer choices will not be from the passage, it will be very difficult to predict what the right answer will look like, so I should go into the answer choices just knowing that I’m looking for some explanation for why our brains would develop in ways that would encourage us to react in the same ways to specific stimuli.
A isn’t a bad candidate for the right answer. I can see how our brains might need to react to shapes and colors (incomplete visual information) because those glimpses might be indicative of the presence of dangerous predators. But that’s a lot of conjecture: enough that I’m too nervous to choose A without eliminating the other answer choices first.
“Vocalizations” immediately makes B wrong, since the passage is primarily focused on color-word synesthesia: synesthesia that the passage claims only involves visual processes.
C is too ambiguous to be correct. Foods might involve visual processes, but “any combination of characteristics” would also likely include smell and taste, which the passage does not link to color-word synesthesia.
D is definitely visual, but it is too visual: does not necessarily involve the interlacing of two different kinds of senses, as synesthesia demands. So D has to be incorrect.
Grapheme is mentioned in paragraph 2: “The experience is described more accurately, then, as a color-grapheme association than as a color-word association.” So that sentence already tells me that a grapheme is not a word. Moving backwards in the sentence, since the quote is phrased in such a way that I know that it is likely that the claim will be supported by preceding arguments, I see that “The link is not between meaning and color, or sound and color, but between the visual appearance of the first letter of the word and color.” So grapheme has something to do with the visual appearance of a letter.
The synesthetic element of a letter is the combination of grapheme and color. Graphemes are not necessarily synesthetic, so A can’t be right.
A syllable usually involves multiple letters, while the passage claims that the grapheme that is the first letter of a word is enough to encourage a synesthetic response, so B has to be wrong.
C is wrong because grapheme is visual, and so would not emerge with a spoken word.
D is vague, but it has to be correct, since “unit” here could refer to letter. I can choose D and move on.
This is a question that the passage had to ask, since the passage has a central term that is difficult to define. I know that the whole passage is about synesthesia, so synesthesia has to be defined early in the passage. It is defined in the first paragraph: “The perception of a color when one hears words is the most common example of the phenomenon of synesthesia. Synesthesia can be visual-tactile, visual-gustatory, tactile-visual, or almost any combination of two senses, but reports are dominated by visually related synesthesias, and olfactory and gustatory synesthesias are less common.” The example of the color and word tells me that synesthesia occurs when something that usually triggers just one sense triggers an unrelated sense. So I’m looking for a choice that describes such an occurrence.
“Thinking” is not a sensory response, so A doesn’t describe two senses being triggered. I can therefore eliminate A.
B, however, does describe two senses triggering when there is only stimulus for one sense. The sight of the word “trumpet” triggers one’s hearing, even when no note is played. That resembles the passage description of color-word synesthesia, so B has to be right.
C has a similar problem to A. Both describe thinking, which is not a sensory response. So C is therefore also wrong.
D also runs into the same problems that A and C do. Despite the way the term sounds, visualization is a mental process, not a sensory one. That means D does not describe two senses being triggered, so D has to be wrong.
I know that the passage argues that formal instruction—which is synonymous with symbolic and derived learning (paragraph 3)—has its weaknesses. I also know that the Louisville study, which is mentioned in the last paragraph, is meant to further illustrate this because the study is prefaced by: “There is growing evidence that while symbolic and derived instruction has positive short-term effects, it may also have negative long-term results.” This question is phrased as if the follow-up study is supposed to weaken this claim because it mentions a benefit of symbolic and derived instruction. But the study does not say that there are no benefits to symbolic and derived instruction; it says instead that benefits will be “short-term.” So I can imagine that, since I need to come up with answers that are based on what the passage argument would say, the author would suggest that the slight advantage in reading skills gained from formal preschool programs is just short-term: that it will lead to “negative long-term results.”I. all girls benefit from early formal education. I doesn’t say this, and it doesn’t make sense as an answer choice because it only tells me about why formal preschool would help some girls: not why “manipulative preschool programs” would be less effective. A correct option needs to account for both—since the author suggests there is something wrong with formal programs and is in favor of manipulative preschool programs—so I has to be wrong.
II. early formal instruction benefits many children and should be universally applied. II has the same problem; it runs counter to the logic of the passage, which talks extensively about the problems with early formal instruction.
III. although formal preschool education appears to benefit girls slightly, the amount is minimal, and concerns such as total literacy levels and other educational goals should take precedence. III is all that’s left and so has to be right, and it is. It matches my prediction, since it claims that gains will be minimal, and that there could be negative long-term results that need to be taken into account.
II is wrong because it advocates for early formal instruction, which the passage argues against.
III is the only correct option, so I can choose it and move on.
I is rhetorically the same claim as II, so it’s also wrong.
And D has to be wrong because I and II are.
I know that the argument in paragraph 2 establishes some of the problems with formal instruction: why it might be more effective in countries that speak phonetic languages, and less effective in nonphonetic language speaking countries. So I can already guess that just as digital clocks are easier to read, phonetic languages are easier to learn. That corresponds with the end of paragraph 2: “In a phonetic language, each symbol has only one sound attached to it, and thus learning this language is easier than learning a language where the same sound can be represented by different letters and the same letter can represent different sounds. In a sense, a phonetic language is to a nonphonetic, or only partially phonetic, language what a digital clock is to a regular clock face.”
A matches my prediction. The end of paragraph 2 states that phonetic languages are easier to learn, and the way that the analogy is phrased at the end of the paragraph suggests that I’m supposed to equate phonetic languages with digital clock faces, so it stands to reason that just as a digital clock is easier to read than a regular clock face, so too is a phonetic language easier to learn than a nonphonetic or partially phonetic language.
The last sentence involves a parallel: phrasing that suggests that the relationship between two terms is the same as the relationship between two other terms. Parallels work through similarity: the first term in the first pairing is meant to be read as similar to the first term in the second pairing, and the second term in one is meant to be read as similar to the second term in the other. B breaks the parallel by making the first term in the first pairing—phonetic languages—the same as the second term in the second pairing: regular clocks. So B has to be wrong.
C also breaks the parallel; this time, the second term of the first pairing (nonphonetic languages) is equated with the first term in the second pairing (digital clocks).
D makes a slightly different error. This time, the correct terms of the parallel are correlated—regular clock faces are meant to be seen as similar to nonphonetic languages—but this time, the passage gets the significance of the relationship incorrect. The passage suggests that just as a regular clock face is harder to read than a digital one, nonphonetic languages are harder to learn than phonetic ones. So I can eliminate D.
Only one study is mentioned in the passage: the Louisville study, in the last paragraph. I know that the study is meant to show that “while symbolic and derived instruction has positive short-term effects, it may also have negative long-term results.” So I can assume that this study would be important for curriculum development because it would help inform curriculum decisions by giving people information regarding both short-term and long-term effects of curriculum decisions.
That doesn’t match my prediction, so I can eliminate it. The study doesn’t talk about when students should begin kindergarten.
B matches my prediction, so I can choose it confidently and move on.
This is incorrect, because the study also talks about “negative long-term results.”
And I can immediately eliminate D because the quote from the last paragraph does not mention it.
The other questions have also dealt heavily with the main idea of the passage, so I should be able to find it quickly. I’m just looking for a choice that mentions that “individual children who are exposed to formal instruction […] too early experience both short-term and long-term negative effects” (paragraph 3).
A is immediately problematic because it’s making a very strong claim—that something has been “fully explored”—that I would definitely have noticed if the passage had mentioned it. I can therefore eliminate A.
B matches my prediction. It encapsulates the beginning of the passage, which points out problems with formalized reading instruction that happens too early, and it aligns with both McCarty’s doctoral dissertation (paragraph 4) and the Louisville experiment (last paragraph). The right answer to a main idea question has to encapsulate the major thrust of the passage, and B clearly does.
C doesn’t mention the problems with early formal education, so it cannot be correct. The passage is not arguing for a plethora of approaches per se; it is focused primarily on establishing the negative implications of a practice. Since C is not, C has to be incorrect.
D has a similar problem to C: it is missing key passage information. The passage is focused more on the problems that can come with gaining literacy in certain ways. D doesn’t talk about different methods of gaining literacy, so D has to be incorrect.
Again, my work on previous questions helps me here. I know that symbolic and derived learning is a direct quote from paragraph 3, where it is stated as another way of saying “formal instruction.” So I’m looking for an answer choice which mentions that, or something like it.
A does not mention anything like what I’m looking for.
Same with B – there’s nothing formal here.
C though definitely includes the word “formal.” Since this is a bit of a quick-and-dirty prediction, though, I should just check to see if D also mentions “formal,” in case I need to make a decision between C and D.
D does not mention “formal instruction,” so I can eliminate it and move on.
This is a reference to the first of two qualities that the author argues are unique to humans in paragraph 2: “An animal without consciousness may ‘know’ the world it inhabits, but only the human animal knows it knows.” So I’m looking for an answer choice that refers to the idea that animals are not self-aware: lack the ability to know that they know or don’t know something.
A works as the right answer because it does not describe knowing factually, but rather the ability to know that one knows something: in this case, to know that one is an individual.
I can eliminate B because it doesn’t necessarily involve the self-awareness that the quote from paragraph 2 implies. One can learn from an experience without being aware that one is learning.
C is wrong for the same reason as B: because it is not necessary to be self-aware to express knowledge. Computers express knowledge all the time, but are not clearly capable of demonstrating that they know that they know something.
D is probably tempting because the phrase “fully aware” sounds like it should include some self-consciousness, but “fully aware” is such a vague concept that it actually makes D less likely to be correct. If the passage does not explicitly explain what it means to be fully aware, how am I to judge whether or not one lacks full awareness if one lacks consciousness? D therefore has to be incorrect.
This question is checking to see if I understand an argument enough to be able to discern what parts of it are necessary: that it to say, which parts of the arguments are such that without them, the argument wouldn’t make sense. The role of culture in human evolution is most directly discussed in the last two paragraphs. My initial reading of the passage showed me that the author brings up human culture in order to explain the rapidity of human change: how quickly human beings change relative to other animals. This is explained in paragraph 5: “When biologists speak of rapid evolutionary change, they mean something on the order of 10,000 years. What has enabled Homo sapiens to speed up the pace of change in its world is culture: Cultural evolution is incomparably faster than genetic evolution.” The author focuses so much on the slowness of biology because paragraph 4 explains that biology is the typical agent of change for animals. If, in other words, biological changes happened more quickly, then it would cast doubt on the idea that cultural evolution contributes to human evolution. So I’m looking for an answer that mentions how slowly biology develops, or how quickly culture develops.
This doesn’t have anything to do with culture, so I can safely eliminate A.
B does mention culture, but B doesn’t talk about human evolution, so there’s no way that B could be a claim on which the argument mentioned in the question stem depends. B is therefore wrong.
C is worse than A and B because it lacks a mention of both culture and evolution.
D, though, mentions both culture (rapid social changes) and biology (genetic mutations). D also matches my prediction, since it mentions that genetic mutations are slow.
I know that the three wrong answers will be examples of cultural evolution, while the right answer could be anything. That means it’s going to be easier to eliminate the wrong answers than it will be to predict what the right answer will look like. So I’m looking for ways that culture encourages changes that fit the passage definition of natural selection, which is stated in paragraph 4: “adaption to shifting circumstances and environments.”
A is a great candidate for the right answer because it does not clearly involve a cause and an effect, which natural selection implies. In the passage description of natural selection, adaption is caused by shifting circumstances and environments. A is just a statement of fact, and implies nothing about how either group of children adapt to a change in circumstances. That makes A correct. However, it is absolutely understandable if this choice does not clearly jump out as correct, since the best strategy for this question is to eliminate the wrong answer choices.
B has to be incorrect because it describes a cause-and-effect situation that resembles natural selection. If natural selection is about adapting to changing circumstances, here we have proof of adaptation (making more violent TV shows) in the face of a change of circumstances (the popularity of violent television shows).
C also has to be incorrect because it implies a similar change in relation to circumstances. During a period of economic decline, social programs become more important, since people are less capable of addressing their needs solely on the basis of their individual economic power. So since there is a change in circumstances (economic decline) that results in some adaptation (a political landscape that becomes friendlier to social programs), C is an example of natural selection via culture.
D also describes the cause-and-effect relationship necessary for the kind of change that would qualify as natural selection. Here, a change in circumstances (a generation creating new terminology) leads to adaptation (the transformation of language). That makes D wrong.
I know from my initial reading of the passage that the question is asking me to assume something that goes against the passage arguments. Specifically, the human ability to be concerned with more than just their immediate surroundings challenges one of the ways the author distinguishes animals and humans in paragraph 3: “within civilization exists an animal with concerns that can be totally divorced from the business of subsistence, an animal with concerns that can be totally divorced from the business of subsistence, an animal with frontiers that can extend beyond its immediate material surroundings, an animal that wants—indeed needs—to discover things about itself and the world it inhabits. This animal has an insatiable appetite for knowledge.” So if humans were actually only capable of paying attention to their immediate surroundings, all of this would be challenged. I’m therefore looking for an answer choice which mentions the lack of something on this list.
I can eliminate A because there is no clear reason why the information in the question stem would suggest A. The assumption in the question stem doesn’t clearly refer to consciousness, so I can’t tell whether or not the assumption would suggest or not suggest A.
I can eliminate B because there’s no obvious way that the assumption in the question stem refers to cultural evolution. It’s more than possible to create culture even if you only pay attention to your immediate surroundings, and there’s nothing in the question stem stopping that culture from outpacing genetic evolution as human culture generally has.
I don’t know what being unable to think beyond one’s immediate surroundings has to do with language, so I can eliminate C.
I would not have chosen D if I hadn’t thought through the question beforehand, and paid attention to the arguments around the part of the passage that the question stem affects. But D has to be right, since the “appetite for knowledge” is tied into the human ability to think beyond the “business of subsistence.”
Language comes up in paragraph 2, as part of the author’s second point regarding what makes humans unique: “The second evolutionary innovation crucial to the unusual course of human history is culture, the facility not only to impose an unprecedented degree of artificiality on the world but to participate in a collective, cumulative learning experience. A vital part of this facility is language, a vehicle for complex thought processes. Through culture, each new generation benefits not only from its own experience and that of its parents but also from the collective wisdom of all previous generation.” Implications can be difficult to predict, so I should just go into the answer choices ready to eliminate answer choices that don’t work as implications of the above passage.
This sounds generally right, since the passage claims that language is a part of culture, but A is making a pretty strong claim: that culture is necessary for language. And the passage does not make a strong enough claim to suggest A. All the passage says is that language is a “vital part” of culture, which could mean that language would not exist without culture, but it doesn’t have to mean that. A supermarket is likely considered vital for selling bread, but it is not as if bread selling is necessary for a supermarket to exist: that supermarkets would not exist if they did not sell bread. Since A is a much stronger claim than the passage makes, A can’t be correct.
I know that the passage argues that humans have culture, but I don’t have a good reference point in the passage to know if animals have language. The second paragraph is too focused on human beings for me to infer whether or not animals have languages, so B can’t be correct.
C has to be wrong because it distorts the passage. Paragraph 2 states that language helps facilitate complex thought processes, but C implies that language is the only way to facilitate complex thought processes. C is therefore wrong for the same reason A is wrong: because it makes a much stronger claim than the passage does.
D is the last answer choice, and so it has to be right. It is not one that I would have chosen, if only because paragraph 2 talks about what culture and language does, but it does not clearly state that the creation of a “cumulative learning experience” and the transmission of “the collective wisdom of all previous generations” is the “primarily important function” of culture and language. It’s possible that both culture and language have other more important functions, but that teaching and the passing on of knowledge are good benefits of both. But again, D is the last choice standing, and I can see how the test-makers arrived at D. Not every answer is going to be ironclad, but as long as it is something I can see could be right, and the other answer choices are clearly wrong, even a poor correct answer choice is still a correct answer choice.
This is a challenging passage, and even this example is pretty challenging. Whenever that is the case, though, I can rely on the passage argument to help me. I know that the last paragraph clearly lays out challenges to the author’s argument in favor of a prototype model of concepts because it begins with “Yet recent findings indicate that in ordinary life, categorization is more a process of inductive inference than of comparison with a prototype.” I also know that the example of the pizza itself, which I definitely find confusing, is not as important as what it is supposed to be an example of: how “categorization is more a process of inductive inference than of comparison with a prototype.” I need to figure out a way to weaken this claim. Since the claim implies that the example of the pizza is inductive and it is different from the prototype model, that implies that the prototype model is not an inference. So my prediction is that the right answer will mention that somehow, the prototype model comes about through inferences, or that the pizza example is not actually an inference.
A large part of the last paragraph is taken up with the author’s support of the claim insofar as the author elaborates heavily on the pizza example, so A can’t be right.
The only way B would be right is if the passage made it very clear that there was some sort of error in thinking here. Since the passage does not imply that, B can’t be right. I need an answer choice which speaks more to the actual claims, and that does not suggest that there is something wrong with the argument as it is written.
C matches my prediction, so I can choose it and move on.
D is immediately wrong because the passage example doesn’t talk about flattened quarters. Changing such a vital part of the example actually means that D is no longer talking about the example given in the passage, so D would not be a valid objection to the claim referenced in the question stem.
Again, what a jumble of words. No wonder the passage can ask a question like this: one that just checks to make sure I understand what the actual text of the passage says. The question quotes the end of paragraph 2, so there is no explanation following to help me understand the phrase, which means all I can do is use the paragraph argument and my reading skills to render the statement in the question stem into common English. “Typicality ratings” here refers to the previous sentence: “Apple and peach are judged to be typical fruits, raisin and fig less typical, and pumpkin and olive atypical.” To say that people’s answers for what they conceptualize most immediately as a fruit are “uncorrelated with their familiarity” has to mean personal familiarity: that what people rate as a typical fruit mostly does not have anything to do with whether or not they are personally familiar with the fruit. In other words, even someone who lived on a pumpkin farm might still conceptualize fruits as apples or peaches. So I’m looking for an answer choice which mentions that these typicality ratings are not purely subjective: that they are not only based on people’s individual experience and preferences.
A is the exact opposite of what the statement in the question stem says. The trick here is that this opposite answer choice involves many of the same words as the statement in the question stem, but “correlate positively” is the opposite of “uncorrelated,” so A can’t be right.
B is actually the same idea as A. If typical instances were encountered more often than atypical instances, then that would suggest that typical instances had something to do with familiarity, since we grow more familiar with what we encounter more often. B is therefore wrong.
I’m already mistrustful of C because “differential knowledge” is not an intuitive term. My best guess is that “differential knowledge” refers to differences between what people know, which again goes back to personal experience. So C has to be wrong.
D matches my prediction, and so I can choose it and move on.
This question refers back to the same paragraph as the last question. Paragraph 2 begins the passage explanation of a prototype model of concepts by explaining that “Apple and peach are judged to be typical fruits, raisin and fig less typical, and pumpkin and olive atypical.” In other words, our understanding of the concept of fruit is most typified by apples and peaches. The end of that paragraph also states that these type ratings are reliable: “uncorrelated with the frequency of occurrence or familiarity of the instances.” So what does it mean when preschoolers in England refer to a less typical fruit instead of a more typical one? That types are not universal. This is raised in paragraph 4: “These typicality effects show that the instances of concepts are not always equal. Yet equality would be expected if, as previously assumed, concepts could be defined in a way that was true of every instance.” In other words, if types were universal, then there might not be any need for a prototype model, because the classical view of concepts—as outlined in the first paragraph—or something like it would better explain how concepts work. So I’m looking for an answer choice which claims that typing may be reliable, but it is not universal.
This is incorrect, because it actually suggests the opposite of what I’m looking for. The classical explanation, as explained in the first paragraph, is meant to be a universal explanation for concepts, while the preschoolers are an example of people departing from a norm.
B matches my prediction, so I can choose it and move on.
I can eliminate C because the preschoolers in the question stem are not clearly going through some deductive process. I would need more information regarding why those preschoolers came to associate fruits with raisins instead of peaches before C could be correct.
D is meant to be attractive because the situation in the question stem is a challenge to a model of categorization, but it is immediately wrong because it’s making a statement which goes further than the passage does. The passage only talks about two models for understanding concepts: the classical model and the prototype model. Just because the prototype model has some failings, that does not tell me enough for me to know about whether every other model for categorization out there is also unable to fully explain categorization.
I know from my initial reading of the passage that a prototype is defined in paragraph 4: “Whether, or how readily, a particular object is classified as a concept member thus depends on its similarity to a hypothetical prototype incorporating all of these properties.” “These properties” are explained in the previous two sentences: “It seems that most natural concepts are defined not by criterial conditions but by properties that are perceptually salient. These properties occur in various numbers and combinations and predominate in only some instances—those considered the ‘best’ examples of the concept.” So if a prototype is the “best” example of a concept, I’m looking for an answer choice that also involves understanding something on the basis of some “best” or central concept.
Since this is talking about language, A would be analogous to the concept of a prototype if it mentioned exemplary words or sentences. “Grammatical structure” is neither; it is more like rules, and so better resembles the classical theory that the prototype model is meant to correct. That makes A incorrect.
B doesn’t seem to suggest that I can understand the brain on the basis of its psychological functions, since it focuses on the fact that the brain is compartmentalized rather than the functions themselves. So B has to be incorrect.
C is a strange one. It does suggest that I can understand planets by understanding smaller bodies, but the fact that these planets “accrete,” or gain new properties as they grow into planets, functions in an opposite way to prototypes. A prototype of a fruit is a prototype because it encompasses the features of a fruit that occur in various quantities and combinations in other fruits. I don’t have to add anything to it in order to better understand other fruits, whereas I need to add something to the smaller bodies mentioned in C to understand planets. That’s enough dissimilarity with the passage to make C wrong.
D is the last answer left, and so it has to be right. If a peach is a prototypical fruit because it contains features that other fruits have in various combinations, then it is as if my understanding of all the various fruits out there derives from this central idea of a peach. That seems to resemble the idea that all species come from a common form, since I can probably understand something about all the other species if I study that original common form.
This is a question that tests how well I understand prototype theory. If someone asks if X is a bird when X is not a bird, then the response time is likely a measure of how long the takes to compare X to the prototype of a bird in order to figure out how similar the two are. The less similar X is to a prototypical bird, the faster it should be rejected. So I’m looking for an answer choice which suggests that.
If anything, a bat should take more time to reject than a mule. The mule lacks many of the features that one would associate with a bird—feathers, wings—while a bat has at least some of those features. It stands to reason that one would need to think about more features of the bat and the prototypical bird to determine that a bat is not a bird than one would a mule, and having to think through more features would involve more time. A is therefore incorrect.
B makes more sense. A mule lacks way more features of a prototypical bird than a bat does, so it should be rejected more quickly.
C is definitely wrong, just because a bat should be rejected more slowly than a mule will.
I have no basis to judge D, since the passage does not talk about how quickly rejection takes place relative to acceptance. So I can eliminate D.